Medicine Long Case Discussions

Medicine Long Case Discussions

These extended patient encounters are designed to mirror real-world clinical practice. They offer invaluable experience in history-taking, physical examination, and logical clinical reasoning. This ultimately fosters the development of well-rounded clinicians capable of managing complex medical cases effectively.

Adult Medicine Long Case: Prolonged Fever (Infective Endocarditis)

Patient Summary

A 50-year-old woman presents with a 4-week history of high-grade, intermittent fevers, associated with chills, anorexia, and significant weight loss. For the past 3 days, she has developed shortness of breath on exertion, difficulty breathing when lying flat (orthopnoea), and episodes of waking from sleep gasping for breath (paroxysmal nocturnal dyspnoea). She has a past history suggestive of acute rheumatic fever in childhood but has not been on regular prophylaxis. On examination, she is in heart failure. Auscultation reveals murmurs of Grade III mitral regurgitation and aortic regurgitation.

How do you structure the history for a patient with prolonged fever?

The history must be meticulous to uncover subtle clues. The main goals are to establish the fever profile and screen for the three main categories of causes: Infections, Neoplasms, and Autoimmune/Inflammatory conditions.

  1. History of Presenting Complaint:
    • Chronology: A precise timeline of the illness.
    • Fever Pattern: Describe the pattern (sustained, intermittent, remittent, relapsing – see Diagram 1 in the text). Note that this is often altered by antipyretics and antibiotics.
    • Associated Symptoms: Document constitutional symptoms like night sweats, weight loss, anorexia, and chills.
  2. Systematic Screening Questions: Systematically ask questions to look for a focus, categorized by etiology.
  3. Contextual History:
    • Origin: Community-acquired, recent hospitalization, or immunocompromised state?
    • Past Medical/Surgical History: Especially valvular heart disease, prosthetic valves, immunosuppressant drugs.
    • Travel and Social History: Foreign travel, high-risk occupations (farming), sexual history, IV drug use.
What specific questions would you ask to screen for Infectious causes?
Infection Type Specific Questions
Tuberculosis Nocturnal fevers, night sweats, chronic cough, hemoptysis, shortness of breath, back pain (Pott’s disease), history of contact with TB.
Abscesses Localized pain (e.g., right upper quadrant pain for liver abscess, pelvic pain/discharge for pelvic abscess), recent surgery or trauma.
Infective Endocarditis History of valvular/congenital heart disease, prosthetic valves, IV drug use. Ask for new features of heart failure.
Brucellosis Exposure to farm animals, consumption of unpasteurized milk, testicular pain, back ache.
Typhoid / Typhus Recent travel to endemic areas, alteration in bowel habits (constipation then diarrhea), skin rash, muscle pain.
Retroviral (HIV) High-risk sexual exposures, IV drug use, history of recurrent opportunistic infections. Fever can be due to the primary infection or a secondary infection.
Malaria Recent travel to malaria-prevalent countries.
What specific questions would you ask for Neoplastic and Inflammatory causes?
Category Diagnosis Specific Questions
Neoplastic Lymphoma “B symptoms” (fever, night sweats, weight loss), pruritus (itching), palpable lumps in the neck, axilla or groin.
Solid Tumors (e.g., Renal Cell Carcinoma) Abdominal pain, hematuria, constitutional symptoms.
Inflammatory / Autoimmune SLE / Vasculitis Inflammatory joint pain, oral ulcers, alopecia, skin rashes (malar rash), frothy urine (proteinuria), seizures. For vasculitis, ask about jaw claudication, visual changes, shoulder/hip pain.
Still’s Disease High spiking fevers, inflammatory joint pain, and a characteristic transient, salmon-pink rash.
What are the key findings on physical examination?

The examination is a detailed search for clues to the underlying diagnosis.

  • General Examination:
    • Look for pallor, icterus, clubbing, and lymphadenopathy.
    • A thorough skin check for rashes (autoimmune), petechiae, or splinter hemorrhages.
    • Look for stigmata of infective endocarditis: Osler’s nodes (painful nodules on fingers/toes), Janeway lesions (painless red macules on palms/soles), splinter hemorrhages.
  • Cardiovascular System:
    • This is critical in this case. Listen carefully for new or changing murmurs.
    • Assess for signs of heart failure (elevated JVP, peripheral edema, basal crepitations).
  • Abdomen: Palpate for hepatosplenomegaly (can be seen in infections like typhoid, HIV, or malignancies like lymphoma).
  • Nervous System: A fundoscopy is essential to look for Roth spots (retinal hemorrhages with pale centers), which are a classic sign of IE.
What investigations would you perform for suspected endocarditis (Duke Criteria)?

The diagnosis is confirmed using the Modified Duke Criteria. The criteria were significantly updated in 2023 (2023 Duke-ISCVID Criteria) to include modern imaging and diagnostic techniques.

1. Microbiological Investigations (Crucial Major Criterion):

  • Blood Cultures: The cornerstone of diagnosis. The recommendation is to draw three sets of blood cultures from different venipuncture sites, with at least 1 hour between the first and last draw, before starting antibiotics. This maximizes the chance of identifying the causative organism.

2. Echocardiogram (Crucial Major Criterion):

  • Transthoracic Echocardiogram (TTE): The first-line imaging test. It is non-invasive and can identify large vegetations. Sensitivity is around 40-60%.
  • Transesophageal Echocardiogram (TOE): A more sensitive test where a probe is passed into the esophagus. It has a much better view of the heart valves and is superior for detecting smaller vegetations, abscesses, and complications with prosthetic valves. A TOE should be considered if the TTE is negative but clinical suspicion remains high.

3. The 2023 Duke-ISCVID Criteria:

A diagnosis of ‘Definite IE’ requires 2 Major criteria, or 1 Major + 3 Minor, or 5 Minor criteria.

Major Criteria Minor Criteria
  1. Blood culture positive for IE: Typical microorganisms from ≥2 separate cultures OR persistently positive cultures.
  2. Evidence of endocardial involvement:
    • Echocardiogram positive for IE (vegetation, abscess, new partial dehiscence of prosthetic valve, new valvular regurgitation).
    • Advanced Imaging Positive: Abnormal activity around a prosthetic valve on PET/CT OR definite paravalvular lesions on cardiac CT.
  1. Predisposing heart condition or IV drug use.
  2. Fever > 38°C.
  3. Vascular phenomena (e.g., major arterial emboli, septic pulmonary infarcts, mycotic aneurysm, Janeway lesions).
  4. Immunologic phenomena (e.g., glomerulonephritis, Osler’s nodes, Roth spots, positive Rheumatoid Factor).
  5. Microbiological evidence that does not meet major criteria.
Source: Fowler VG, et al. The 2023 Duke-ISCVID International Criteria for Infective Endocarditis. Clinical Infectious Diseases. 2023.
What are the causes of culture negativity in IE?

Up to 30% of IE cases can be culture-negative. The reasons include:

  • Prior antibacterial therapy: This is the most common cause.
  • Infection with fastidious organisms: These are slow-growing or require special culture media. The classic group is the HACEK organisms (Haemophilus, Aggregatibacter, Cardiobacterium, Eikenella, Kingella).
  • Infection with intracellular organisms: Such as Coxiella burnetii (Q fever) or Bartonella, which require serological tests for diagnosis.
  • Fungal endocarditis.
  • Improper collection technique.
Discuss the principles of antibiotic therapy in IE.

Antibiotic therapy is the mainstay of treatment. It must be bactericidal and prolonged (typically 4-6 weeks).

Empirical Therapy (Before culture results are known):

The choice depends on the clinical setting. The advice in the textbook (Penicillin + Gentamicin) covers streptococci and provides synergy for enterococci but may miss Staph. aureus, a common cause.

  • Community-Acquired Native Valve Endocarditis (Patient stable): A modern regimen would be IV Amoxicillin + IV Gentamicin. This covers viridans group streptococci, enterococci, and HACEK organisms.
  • Severe Sepsis or Healthcare-Associated IE: A broader regimen is needed to cover MRSA. A common choice is IV Vancomycin + IV Gentamicin.

Directed Therapy: Once culture and sensitivity results are available, the antibiotic regimen should be tailored to the specific organism.

Source: NICE Guideline [NG188] Sepsis: recognition, diagnosis and early management (2017) and ESC Guidelines for the management of endocarditis (2023).
When would you consider a cardiothoracic surgical opinion?

Surgery is required in about 50% of patients with IE. The decision is urgent and should be made by a multidisciplinary “Endocarditis Team.” The main indications fall into three categories:

  1. Heart Failure:
    • This is the most common and urgent indication.
    • Caused by severe acute aortic or mitral regurgitation, or valve obstruction.
  2. Uncontrolled Infection:
    • Persistently positive blood cultures despite appropriate antibiotic therapy.
    • Infection with organisms that are difficult to eradicate (fungi, resistant bacteria).
    • Local complications like abscess, pseudoaneurysm, or fistula formation.
  3. Prevention of Embolism:
    • Presence of large vegetations (>10 mm) with a prior embolic event.
    • Very large vegetations (>15 mm) even without an embolic event, especially on the mitral valve.
Source: 2023 ESC Guidelines for the management of endocarditis.
What is the current advice on antibiotic prophylaxis for IE?

This is a major change from older practice.

  • Routine antibiotic prophylaxis for dental procedures is no longer recommended for most patients at risk of IE in the UK.
  • The focus has shifted to maintaining excellent oral and dental hygiene to reduce the risk of bacteremia from daily activities like toothbrushing.
  • However, prophylaxis should still be considered for patients at high risk of IE (e.g., those with a previous episode of IE, prosthetic heart valves, or certain types of congenital heart disease) who are undergoing high-risk dental procedures.
Source: NICE guideline [CG64] Prophylaxis against infective endocarditis (2016).

Adult Medicine Long Case: Prolonged Fever (Lymphoma)

Patient Summary

A 40-year-old businessman presents with prolonged fever for 3 weeks. He notes the fever is typically worse in the afternoons and is associated with drenching night sweats, significant anorexia, and an unintentional weight loss of 10kg. He also complains of generalized fatigue and malaise. On examination, he has generalized, non-tender, rubbery lymphadenopathy in the cervical and axillary chains. His abdomen is soft, with a palpable liver and a spleen tipped felt on inspiration.

What is the most likely diagnosis, and why do you think so?

The most likely diagnosis is Lymphoma.

Lymphoma is a top differential in any patient with Pyrexia of Unknown Origin (PUO), especially when the following features are present:

  • Constitutional “B” Symptoms: This is the classic triad of:
    1. Unexplained fever (>38°C).
    2. Drenching night sweats.
    3. Unintentional weight loss (>10% of body weight in 6 months).
    This patient presents with all three.
  • Generalized Lymphadenopathy: The presence of enlarged lymph nodes in multiple, non-contiguous areas is highly suspicious.
  • Organomegaly: Hepatosplenomegaly is a common finding due to infiltration of the liver and spleen by lymphoma cells.

What are the close differential diagnoses?

The clinical picture of B symptoms, lymphadenopathy and organomegaly can also be caused by:

  • Disseminated Tuberculosis (TB): Miliary TB or TB lymphadenitis can mimic lymphoma perfectly.
  • Retroviral infection (HIV): Can present with fever, lymphadenopathy, and weight loss, either from the virus itself or an opportunistic infection.
  • Sarcoidosis: An inflammatory condition that can cause lymphadenopathy and constitutional symptoms.
How would you investigate this patient to confirm the diagnosis?

The absolute most important investigation is to obtain tissue for histology.

  1. Histological Confirmation (The Gold Standard):
    • Excisional Lymph Node Biopsy: This is the definitive investigation. A whole lymph node is surgically removed (excised), preferably from a palpable, superficial site (e.g., cervical, axillary). This allows the pathologist to assess the node architecture, which is crucial for diagnosis and subtyping. A fine-needle aspiration (FNA) or core biopsy is less ideal as it may not provide enough tissue.
  2. Staging Investigations (to determine the extent of disease):
    • CT Scan: A contrast-enhanced CT of the Chest, Abdomen, and Pelvis is essential to evaluate the extent of lymphadenopathy (mediastinal, retroperitoneal) and organ involvement.
    • PET-CT Scan: This is now the standard of care for staging in many types of lymphoma (especially Hodgkin’s). It is highly sensitive for detecting active disease sites and is also used to assess response to treatment.
    • Bone Marrow Examination: An aspirate and trephine biopsy is performed to check for bone marrow infiltration by lymphoma cells. This is a critical part of staging.
  3. Baseline Blood Tests:
    • Full Blood Count (FBC): May show anemia, or other cytopenias if the marrow is involved.
    • ESR/CRP: Inflammatory markers are usually elevated.
    • LDH (Lactate Dehydrogenase): This is an important prognostic marker. Higher levels indicate a larger tumor burden and a poorer prognosis.
    • Liver and Renal Function Tests.
    • HIV and Hepatitis serology.
What is the difference between Hodgkin’s and Non-Hodgkin’s Lymphoma?

This is a fundamental classification based on the histology of the lymph node biopsy.

Feature Hodgkin’s Lymphoma (HL) Non-Hodgkin’s Lymphoma (NHL)
Characteristic Cell Presence of the large, malignant Reed-Sternberg cell. No Reed-Sternberg cells. A heterogeneous group of cancers.
Cell of Origin B-cell origin. Majority are B-cell origin, but can also be T-cell or NK-cell origin.
Pattern of Spread Typically starts in a single lymph node group and spreads in an orderly, contiguous fashion to adjacent nodes. Spreads in a more haphazard, non-contiguous fashion. Often widespread at diagnosis.
Nodal Involvement Commonly involves nodes above the diaphragm (cervical, mediastinal). Can involve any lymph nodes.
Extranodal Spread Less common. Common. Can involve sites like the GI tract, skin, and CNS.
Prognosis Generally has a very high cure rate with modern chemotherapy +/- radiotherapy. Prognosis is more variable and depends heavily on the specific subtype (indolent vs. aggressive) and stage.

Adult Medicine Long Case: Shortness of Breath (Heart Failure)

Patient Summary

Mrs. D, a 68-year-old housewife, presents with progressively worsening shortness of breath over the past six months. Her breathlessness is now severe, occurring after walking just 50 meters. She reports waking up at night gasping for air (paroxysmal nocturnal dyspnoea) and needing to sleep propped up on several pillows (orthopnoea). This was associated with a cough producing scanty, pinkish sputum. She has a 5-year history of hypertension with poor medication compliance and was treated for a heart attack 6 months ago. On examination, she is in severe respiratory distress, using her accessory muscles. Her pulse is 90/min and regular, respiratory rate is 28/min. Her cardiac apex is displaced to the 6th intercostal space lateral to the mid-clavicular line. Auscultation reveals diffuse bilateral fine crepitations in the lung fields.

Why do you say this patient has heart failure?

The diagnosis of heart failure is made based on a combination of characteristic symptoms, specific signs, and evidence of an underlying structural or functional cardiac abnormality.

Evidence from Symptoms:

  • Progressive Dyspnoea on Exertion: This is a cardinal symptom of left ventricular failure, indicating the heart cannot increase its output to meet metabolic demands.
  • Orthopnoea & PND: These are highly specific symptoms for heart failure. They are caused by the redistribution of fluid from the lower limbs to the central circulation upon lying down, overwhelming the failing left ventricle and causing pulmonary congestion.
  • Pinkish Sputum: Suggests acute pulmonary oedema, a life-threatening manifestation of decompensated heart failure.

Evidence from Signs:

  • Respiratory Distress: Tachyapnoea and use of accessory muscles indicate severe pulmonary congestion.
  • Displaced Cardiac Apex: This is a sign of cardiomegaly (an enlarged heart), a key indicator of underlying structural heart disease (likely left ventricular dilatation).
  • Diffuse Bilateral Crepitations: This is the sound of fluid in the alveoli, a direct sign of pulmonary oedema.

Evidence from Risk Factors:

  • Poorly Controlled Hypertension: A major cause of heart failure through pressure overload leading to LV hypertrophy and eventually failure.
  • Previous Myocardial Infarction: Ischaemic heart disease is the most common cause of heart failure in the developed world. The previous MI has likely damaged the heart muscle, impairing its ability to pump effectively.
How would you establish and investigate a diagnosis of heart failure?

Investigations aim to confirm the diagnosis, identify the underlying aetiology, assess severity, and guide management.

1. Biomarkers (Natriuretic Peptides):

  • This is the recommended first step for a suspected new diagnosis of heart failure.
  • Measure serum BNP (B-type natriuretic peptide) or NT-proBNP. These are hormones released by the heart muscle in response to being stretched.
  • A normal level makes a diagnosis of heart failure highly unlikely. A high level supports the diagnosis and indicates the need for further investigation with an echocardiogram.

2. Echocardiogram (The Key Diagnostic Test):

  • This is essential to confirm the diagnosis. It provides crucial information:
    • Left Ventricular Ejection Fraction (LVEF): Quantifies systolic function. This is critical for classifying the type of heart failure.
    • Structural Abnormalities: Identifies chamber dilatation, LV hypertrophy, regional wall motion abnormalities (from her previous MI), and valvular heart disease.
    • Diastolic Function: Can assess for diastolic dysfunction, the cause of Heart Failure with Preserved Ejection Fraction (HFpEF).

3. Initial Investigations:

  • ECG: A 12-lead ECG is essential. It can show evidence of a previous MI (e.g., Q waves), left ventricular hypertrophy, arrhythmias (like atrial fibrillation), or conduction abnormalities. A completely normal ECG makes systolic heart failure less likely.
  • Chest X-ray: Can show the “ABCDE” of heart failure: Alveolar oedema (bat’s wings), Kerley B lines, Cardiomegaly, upper lobe Diversion, and pleural Effusions.
  • Baseline Bloods: FBC (to exclude anaemia as a cause/exacerbating factor), Urea & Electrolytes (essential for guiding diuretic and ACEi/ARB/MRA therapy), LFTs, and HbA1c.
Source: NICE Guideline [NG106] Chronic heart failure in adults: diagnosis and management (2018).
How do you classify heart failure and why is it important?

Classification is essential as it dictates the evidence-based management strategy.

1. Classification based on Ejection Fraction (LVEF):

  • Heart Failure with Reduced Ejection Fraction (HFrEF): LVEF ≤ 40%. This is the group with the most evidence for mortality-reducing therapies.
  • Heart Failure with Mildly Reduced Ejection Fraction (HFmrEF): LVEF 41-49%.
  • Heart Failure with Preserved Ejection Fraction (HFpEF): LVEF ≥ 50%. Often associated with diastolic dysfunction. Management is primarily focused on treating comorbidities.

2. Classification based on Symptoms (NYHA – New York Heart Association):

  • Class I: No limitation of physical activity. Ordinary activity does not cause symptoms.
  • Class II: Slight limitation of physical activity. Comfortable at rest, but ordinary activity results in symptoms.
  • Class III: Marked limitation of physical activity. Comfortable at rest, but less than ordinary activity causes symptoms.
  • Class IV: Unable to carry on any physical activity without discomfort. Symptoms are present even at rest. (This patient is currently in Class IV).
How would you manage this patient’s Heart Failure with Reduced Ejection Fraction (HFrEF)?

Management has two phases: treating the acute decompensation and initiating long-term disease-modifying therapy.

Management of Acute Pulmonary Oedema:

  1. Sit the patient upright.
  2. Administer high-flow oxygen to maintain SpO2 >94%.
  3. Give IV loop diuretics (e.g., IV Furosemide) to offload fluid.
  4. Administer vasodilators (e.g., IV nitrates) if blood pressure is adequate.
  5. Consider CPAP/NIV if respiratory distress is severe.

Long-Term Pharmacological Management (The Four Pillars):

Modern management for HFrEF is based on initiating four classes of drugs that have been proven to reduce mortality and hospitalisations. These should be started and up-titrated once the patient is stable.

  1. ACE inhibitor (ACEi) or Angiotensin Receptor-Neprilysin Inhibitor (ARNI):
    • Start with an ACEi (e.g., Ramipril). The current recommendation is to switch to an ARNI (Sacubitril/Valsartan) if symptoms persist, as it is superior.
  2. Beta-blocker:
    • Use an evidence-based beta-blocker (e.g., Bisoprolol, Carvedilol). Must be started at a low dose in stable patients and titrated up slowly.
  3. Mineralocorticoid Receptor Antagonist (MRA):
    • (e.g., Spironolactone or Eplerenone). Requires careful monitoring of potassium and renal function.
  4. SGLT2 inhibitor:
    • (e.g., Dapagliflozin or Empagliflozin). This is a major update. These drugs are now recommended for all patients with HFrEF, regardless of whether they have diabetes.

General Management: Includes patient education, lifestyle advice (salt/fluid restriction), smoking cessation, and referral to cardiac rehabilitation.

Source: 2023 ESC Guidelines for the management of acute and chronic heart failure.

Adult Medicine Long Case: Chest Pain (Chronic Coronary Syndrome & ACS)

Patient Summary (Part 1 – Stable)

Mr. Y, a 45-year-old man with a background of Hypertension and Type 2 Diabetes Mellitus, presents with a 4-week history of central chest pain. He describes the pain as a “tightening” feeling that radiates to his left arm and both sides of his jaw. The pain occurs exclusively on exertion, typically after walking about 200 meters, and is completely relieved within a few minutes of resting. He has no associated shortness of breath. His examination is unremarkable, with a BP of 170/80 and normal heart sounds.

What is the diagnosis in the initial presentation and how would you confirm it?

The initial diagnosis is Chronic Stable Angina, now more commonly referred to as Chronic Coronary Syndrome (CCS). The history is classical: exertional, characteristic site and radiation, and relief with rest.

Confirmation of Diagnosis (Modern Approach):

The diagnostic pathway has shifted from functional testing to anatomical imaging as the first line.

  1. Initial Tests: All patients should have a resting 12-lead ECG (which may be normal), FBC, U&Es, LFTs, HbA1c, and a lipid profile.
  2. First-Line Diagnostic Imaging:
    • CT Coronary Angiography (CTCA) is the recommended first-line investigation for patients with typical or atypical angina symptoms in whom the diagnosis is uncertain.
    • It is a non-invasive scan that directly visualizes the coronary arteries. It has a very high negative predictive value, meaning a normal CTCA effectively rules out significant coronary artery disease (CAD).
  3. Non-invasive Functional Imaging:
    • These tests are used if CTCA is inconclusive or contraindicated, or to assess the functional significance of a known stenosis. They aim to detect inducible myocardial ischaemia.
    • Options include: Myocardial Perfusion Scintigraphy (MPS), Stress Echocardiography, or Cardiovascular MRI with perfusion.
  4. Invasive Coronary Angiography:
    • This remains the gold standard for defining coronary anatomy and is performed when revascularization (PCI or CABG) is being considered, or if symptoms are severe despite medical therapy.
Source: NICE Guideline [CG95] Chest pain of recent onset: assessment and diagnosis (2016).
How would you manage a patient with Chronic Coronary Syndrome?

Management is comprehensive, focusing on lifestyle, symptom relief, and secondary prevention to improve prognosis.

1. Lifestyle Modifications and Risk Factor Management:

  • Smoking cessation, dietary advice, regular exercise, weight management.
  • Optimal control of blood pressure and diabetes.

2. Pharmacological Management:

  • For Symptom Relief (Anti-anginals):
    • Immediate Relief: Sublingual Glyceryl Trinitrate (GTN) spray or tablets to use as needed.
    • First-Line Maintenance: Offer either a Beta-blocker or a Calcium Channel Blocker (CCB) based on comorbidities and patient preference.
    • Second-Line: If symptoms are not controlled, use a Beta-blocker and a CCB in combination.
    • Third-Line: If symptoms persist, consider adding a long-acting nitrate, ivabradine, nicorandil, or ranolazine.
  • For Secondary Prevention (to improve prognosis):
    • Antiplatelet therapy: Low-dose Aspirin (75mg daily). (Clopidogrel if aspirin intolerant).
    • Lipid modification: High-intensity Statin (e.g., Atorvastatin 80mg).
    • ACE inhibitor: Consider for all patients, especially those with hypertension, diabetes, or LV systolic dysfunction.

Discussion Point (Part 2 – Acute)

While on therapy, this patient presents to the emergency department with an acute onset of severe, central chest pain that started 4 hours ago. The pain is associated with intense sweating and several episodes of vomiting. His ECG shows ST-segment elevation in leads V1-V6.

The patient now presents with an acute STEMI. How do you manage this emergency?

The diagnosis is an Acute Anterolateral ST-Elevation Myocardial Infarction (STEMI). This is a medical emergency requiring immediate reperfusion therapy.

Immediate Management (Pre-hospital and Emergency Department):

  1. Aspirin 300mg oral loading dose immediately.
  2. Pain Relief: IV Morphine given with an IV anti-emetic (e.g., metoclopramide).
  3. Oxygen: Only if the patient is hypoxic (SpO2 < 94%). Routine oxygen is not beneficial.
  4. Second Antiplatelet: A P2Y12 inhibitor loading dose should be given. In a STEMI patient proceeding to PCI, Prasugrel or Ticagrelor are preferred over Clopidogrel due to their faster onset and greater potency.
  5. Anticoagulation: Administer an anticoagulant (e.g., unfractionated heparin, LMWH).

Reperfusion Strategy: The goal is to open the blocked artery as quickly as possible.

  • Primary Percutaneous Coronary Intervention (PPCI): This is the gold standard treatment. It involves an immediate invasive coronary angiogram and ballooning/stenting of the culprit artery. This should be performed within 120 minutes of first medical contact. The patient needs urgent transfer to a 24/7 heart attack centre.
  • Thrombolysis (Fibrinolysis): This is used if timely PPCI is not achievable. A fibrinolytic drug (e.g., Tenecteplase, Alteplase) is given intravenously to dissolve the clot. The target is a “door-to-needle” time of within 30 minutes of hospital arrival.
Source: 2023 ESC Guidelines for the management of acute coronary syndromes.
How would you know that thrombolysis has worked? What if it fails?

It is vital to assess for successful reperfusion after thrombolysis.

Signs of Successful Thrombolysis:

  • Resolution of Chest Pain: The patient’s pain should significantly diminish.
  • ECG Changes: There should be a >50% resolution of the maximal ST-segment elevation on the ECG taken 60-90 minutes after starting thrombolysis.
  • Reperfusion Arrhythmias: A brief, non-sustained ventricular arrhythmia may occur as the vessel opens.

Management if Thrombolysis Fails:

  • If there are no signs of successful reperfusion after 90 minutes, the patient is considered to have “failed thrombolysis”.
  • This is a clinical emergency. The next step is to arrange for urgent transfer to a PCI-capable hospital for Rescue PCI.
  • Rescue PCI is an immediate coronary angiogram and intervention to mechanically open the artery.

Adult Medicine Long Case: Bronchial Asthma

Patient Summary

Mr P, a 21-year-old student with asthma diagnosed at age 5, presents with an acute onset of severe difficulty in breathing after cleaning his study room. He is in obvious distress, unable to complete sentences. His asthma has been poorly controlled; he only uses a salbutamol (blue) inhaler and experiences symptoms 2-3 times per week and is woken by symptoms twice a month. He has had four exacerbations in the last year. His father also has asthma, and he has a history of eczema. Examination reveals respiratory distress with intercostal recessions and nasal flaring. His respiratory rate is 28/min, pulse is 110/min, and there are diffuse wheezes and few crepitations on auscultation.

How would you assess the severity of this exacerbation?

The severity of an asthma exacerbation is classified clinically to guide immediate management. This patient has features of an acute severe attack.

Moderate Acute Severe Life-Threatening
  • Increasing symptoms
  • PEFR >50-75% best/predicted
  • No features of severe asthma
Any one of:
  • PEFR 33-50% best/predicted
  • Respiratory rate ≥25/min
  • Heart rate ≥110/min
  • Inability to complete sentences in one breath
Any one feature in a patient with severe asthma:
  • Altered conscious level (confusion, drowsiness)
  • Exhaustion, poor respiratory effort
  • Arrhythmia or hypotension
  • Silent chest
  • Cyanosis
  • SpO2 <92%
  • PEFR <33% best/predicted
Source: NICE Guideline [NG80] Asthma: diagnosis, monitoring and chronic asthma management.
How would you manage this acute severe exacerbation?

This is a medical emergency requiring immediate, structured management.

  1. Initial Assessment (ABCDE):
    • Sit the patient upright to ease breathing.
    • Oxygen: Start high-flow oxygen via a non-rebreather mask to maintain SpO2 between 94-98%.
  2. Bronchodilators:
    • Nebulized Salbutamol (5mg) driven by oxygen.
    • Add Nebulized Ipratropium Bromide (0.5mg). The combination is more effective than salbutamol alone.
    • These can be given back-to-back if the patient is not responding.
  3. Systemic Corticosteroids:
    • Administer steroids immediately to reduce airway inflammation. The effect takes 4-6 hours.
    • Options: Oral Prednisolone 40-50mg daily (if the patient can swallow) OR IV Hydrocortisone 100mg.
  4. Monitoring: Continuously monitor clinical parameters (RR, HR), SpO2, and repeat PEFR 15-30 minutes after starting treatment.
The patient does not improve. What will you do now?

If the patient does not respond to initial therapy or develops life-threatening features, senior help (e.g., medical registrar, ICU) must be called immediately while escalating treatment.

Escalation Steps:

  1. Intravenous (IV) Magnesium Sulphate:
    • Give a single slow IV bolus of 1.2-2g of magnesium sulphate. It acts as a bronchodilator.
  2. Arterial Blood Gas (ABG):
    • An ABG is essential to assess for life-threatening features. The “blood gas markers of a life-threatening attack” are:
      • Severe hypoxia: PaO2 < 8 kPa.
      • Normal or high PaCO2: A normal (4.6-6.0 kPa) or rising PaCO2 is an ominous sign of impending respiratory failure and exhaustion. Initially, a patient should be breathing fast and have a low PaCO2.
      • Low pH (acidosis): Indicates respiratory failure.
  3. Further Therapies:
    • IV aminophylline or IV salbutamol can be considered under expert guidance in an HDU/ICU setting with cardiac monitoring due to risks of arrhythmias.
  4. Ventilatory Support:
    • If the patient is exhausting or has worsening blood gases despite maximal therapy, they may require ventilatory support. Non-invasive ventilation (NIV) may be tried, but invasive mechanical ventilation is often necessary.
How would you manage this patient in the long term?

This patient’s exacerbation was a direct result of undertreatment and poor long-term control. The entire approach to his asthma needs to be reviewed.

The GINA 2023 guidelines have made a major shift: SABA-only treatment is NO longer recommended. All adults and adolescents with asthma should receive an Inhaled Corticosteroid (ICS)-containing controller medication to reduce the risk of severe exacerbations.

Long-Term Management Plan:

  1. Patient Education:
    • Explain the nature of asthma as a chronic inflammatory condition.
    • Differentiate between “reliever” (SABA) and “preventer” (ICS) inhalers. Explain that overuse of the reliever is a sign of poor control and that the preventer must be taken every day.
    • Inhaler Technique: This is crucial. Observe his technique with a placebo device and correct any errors. Ensure he uses a spacer device.
    • Action Plan: Provide a written, personalized asthma action plan so he knows how to recognize worsening symptoms and when to increase his medication or seek help.
  2. Pharmacological Therapy (Step-wise Approach):
    • This patient has symptoms more than twice a week, indicating at least moderate persistent asthma. He should be started on Step 3 of the GINA guidelines.
    • Recommended Treatment: A low-dose ICS-LABA combination inhaler (e.g., Symbicort, Fostair) used for both daily maintenance and as a reliever (this is known as MART – Maintenance And Reliever Therapy). This single inhaler approach improves compliance and ensures the patient receives an anti-inflammatory dose whenever they feel symptoms.
  3. Identify and Manage Triggers and Comorbidities:
    • Triggers: Advise on dust mite avoidance measures. Address passive smoking exposure.
    • Comorbidities: Screen for and treat associated conditions like allergic rhinitis and GORD.
  4. Follow-up: Regular follow-up (e.g., in 1-3 months) is essential to assess control, check adherence, and adjust therapy as needed.
Source: Global Initiative for Asthma (GINA) Report, 2023.

Adult Medicine Long Case: COPD

Patient Summary

Mr. P, a 56-year-old laborer and heavy smoker (15 pack-years), presents with an acute episode of severe shortness of breath, from which he has now recovered. He has a history of monthly similar episodes starting from age 48, with worsening symptoms when his sputum turns yellow. He notes progressive breathlessness on exertion but denies any sudden nocturnal episodes. On examination, he appears wasted, is propped up in bed, and uses pursed-lip breathing. He has bilateral ankle edema and a parasternal heave. Auscultation reveals bilateral rhonchi with a patch of bronchial breathing in the left lower zone. There is also mild tender hepatomegaly.

What is your diagnosis and why do you say so?

The diagnosis is an Acute Exacerbation of Chronic Obstructive Pulmonary Disease (COPD), complicated by a likely left lower lobe pneumonia and features of cor pulmonale (right heart failure).

Features Suggesting COPD over Asthma:

  • Age of Onset: Symptoms started after age 40, which is typical for COPD.
  • Strong Smoking History: This is the most important risk factor for COPD.
  • Nature of Symptoms: The patient describes progressive, persistent breathlessness on exertion, which is characteristic of COPD. Asthma typically has more intermittent and variable symptoms.
  • Sputum Production: Chronic productive cough is a common feature. Worsening sputum purulence often signals an exacerbation.
  • Lack of Diurnal Variation: He denies nocturnal symptoms, which are a hallmark of poorly controlled asthma.
  • Examination Findings: Wasting, signs of chronic hypoxemia, and features of cor pulmonale (ankle edema, parasternal heave, hepatomegaly) point towards advanced, long-standing COPD.
How will you evaluate this patient to confirm the diagnosis of COPD?

Investigations are done to confirm the diagnosis, assess severity, and identify complications.

  1. Spirometry (Lung Function Testing):
    • This is the gold standard for diagnosing COPD.
    • It must be performed when the patient is clinically stable.
    • The key diagnostic finding is a post-bronchodilator FEV1/FVC ratio of less than 0.70. This confirms persistent airflow limitation that is not fully reversible.
    • The FEV1 value (as a percentage of predicted) is then used to grade the severity of airflow limitation (GOLD 1-4).
  2. Chest X-ray (CXR):
    • May show features of hyperinflation (flattened hemidiaphragms, increased lung lucency, large retrosternal air space), bullae, or prominent pulmonary arteries (suggesting pulmonary hypertension).
    • It is also crucial for identifying other pathologies, such as the consolidation in the left lower zone in this patient.
  3. Arterial Blood Gas (ABG):
    • Essential during an acute exacerbation to assess for hypoxemia and hypercapnia (CO2 retention), which would indicate Type 2 Respiratory Failure.
  4. Full Blood Count (FBC): To look for polycythemia (a high hemoglobin level secondary to chronic hypoxia) or anemia (which can worsen breathlessness).
How would you manage an acute exacerbation of COPD?

Management of an AECOPD is structured and aimed at reversing bronchospasm, treating the trigger, and ensuring adequate oxygenation without worsening CO2 retention.

  1. Controlled Oxygen Therapy:
    • This is a critical first step. The goal is to correct hypoxemia without knocking out the hypoxic respiratory drive in patients prone to CO2 retention.
    • Start with low-concentration oxygen (e.g., 24% or 28% via a Venturi mask) to achieve a target oxygen saturation of 88-92%.
  2. Bronchodilators:
    • Give nebulized short-acting bronchodilators: Salbutamol 5mg (a beta-2 agonist, SABA) and Ipratropium Bromide 0.5mg (a short-acting muscarinic antagonist, SAMA).
  3. Systemic Corticosteroids:
    • Give a course of Oral Prednisolone 30-40mg daily for 5 days. This has been shown to shorten recovery time and reduce the risk of relapse. IV Hydrocortisone can be used if the patient cannot take oral medication.
  4. Antibiotics:
    • Antibiotics are indicated if there are features of a bacterial infection, i.e., an increase in sputum purulence, plus either an increase in sputum volume or an increase in breathlessness.
    • First-line choice is typically oral Amoxicillin or a macrolide (e.g., Clarithromycin).
  5. Consider NIV (Non-Invasive Ventilation):
    • If the patient has an acute respiratory acidosis (pH < 7.35 and PaCO2 > 6.0 kPa) despite initial medical therapy, NIV should be started urgently. It is proven to reduce mortality and the need for intubation.
Source: GOLD (Global Initiative for Chronic Obstructive Lung Disease) 2024 Report; NICE Guideline [NG115] Chronic obstructive pulmonary disease (acute exacerbation).
How will you manage this patient in the long term?

Long-term management is focused on reducing symptoms, preventing exacerbations, and improving quality of life.

1. Non-Pharmacological Management (Crucial):

  • Smoking Cessation: This is the single most effective intervention to slow the progression of COPD. Offer support and pharmacotherapy (e.g., nicotine replacement therapy, varenicline).
  • Pulmonary Rehabilitation: A structured, supervised program of exercise, education, and psychosocial support. It is one of the most effective treatments for improving breathlessness and exercise capacity.
  • Vaccinations: Offer annual influenza vaccination and a one-off pneumococcal vaccination.

2. Pharmacological Management (based on GOLD ABCD assessment):

The choice of inhaler therapy is based on the patient’s symptom burden (using mMRC or CAT scores) and their history of exacerbations.

  • Initial Therapy: Given this patient has frequent exacerbations and significant symptoms, he would be in Group E (Exacerbators) of the new GOLD classification.
  • Recommended Inhaler: The initial recommended therapy for Group E is a combination of a LAMA + LABA (e.g., Tiotropium/Olodaterol).
  • Adding ICS: If the patient continues to have exacerbations on LAMA/LABA, an Inhaled Corticosteroid (ICS) can be added to make a triple therapy inhaler (LAMA/LABA/ICS). The presence of high blood eosinophil counts makes a response to ICS more likely.

3. Long-Term Oxygen Therapy (LTOT):

  • This is considered for patients with severe, chronic hypoxemia at rest.
  • Criteria: PaO2 < 7.3 kPa, or PaO2 between 7.3-8.0 kPa with evidence of pulmonary hypertension, peripheral edema, or polycythemia.
  • It must be used for at least 15 hours per day to provide a survival benefit. It cannot be prescribed for patients who continue to smoke due to the fire risk.
Source: GOLD 2024 Report.

Adult Medicine Long Case: Interstitial Lung Disease (ILD)

Patient Summary

Mr. B, a 61-year-old labourer, presents with progressive shortness of breath on exertion for 3 months. Initially noticed only during work, it has worsened to the point where he is breathless after walking about 50 meters. He has an associated dry cough with whitish sputum. He denies any chest pain, orthopnoea, or PND. He is a non-smoker with no significant past medical history, drug history, or chemical exposures. On examination, he looks unwell and is propped up. He has grade II finger clubbing and bilateral ankle oedema. His respiratory rate is 20/min, and auscultation reveals fine, end-inspiratory “Velcro-like” crepitations at both lung bases. Cardiac examination shows an elevated JVP, a left parasternal heave, and a loud P2, suggestive of pulmonary hypertension.

What are your reasons for diagnosing Interstitial Lung Disease?

The diagnosis of ILD is strongly suggested by a constellation of clinical features in this patient:

  • Progressive Dyspnoea: The insidious onset and progressive nature of his shortness of breath is the hallmark symptom of chronic fibrotic lung disease.
  • Associated Dry Cough: A non-productive cough is very common in ILD.
  • Fine End-Inspiratory Crepitations: The presence of bilateral, basal, “Velcro-like” crackles is the most important examination finding. This sound is generated by the sudden opening of collapsed small airways surrounded by fibrotic tissue.
  • Finger Clubbing: This is a key physical sign, strongly associated with some forms of ILD, particularly Idiopathic Pulmonary Fibrosis (IPF).
  • Exclusion of Other Causes: The absence of features of heart failure (like PND), obstructive airways disease, or infection points towards an interstitial process.
  • Signs of Complication: The features of pulmonary hypertension (elevated JVP, parasternal heave, loud P2) are a common complication of advanced fibrotic ILD.
What is the aetiology? How do you classify ILD?

ILD is a large, heterogeneous group of over 200 disorders. The first step in classification is to separate cases with a known cause from those that are idiopathic.

1. ILD of Known Cause or Association:

  • Connective Tissue Diseases: ILD is a common complication of conditions like Rheumatoid Arthritis, Systemic Sclerosis (Scleroderma), and Myositis. (Need to ask about joint pain, rashes, Raynaud’s).
  • Occupational/Environmental Exposures (Pneumoconiosis): A detailed occupational history is vital.
    • Asbestosis: From asbestos exposure (shipbuilding, construction).
    • Silicosis: From silica dust (mining, stone cutting).
  • Hypersensitivity Pneumonitis: An immune reaction to inhaled organic antigens.
    • Farmer’s Lung: From moldy hay.
    • Bird Fancier’s Lung: From avian proteins in bird droppings. (Need to ask about exposure to birds/hay).
  • Drug-Induced ILD: Many drugs can cause lung fibrosis. Common culprits include Amiodarone, Methotrexate, and Nitrofurantoin.

2. Idiopathic Interstitial Pneumonias (IIPs):

When no cause is found, the ILD is classified based on its histological and radiological (HRCT) pattern.

  • Idiopathic Pulmonary Fibrosis (IPF): The most common and most severe form of IIP. It has a characteristic HRCT pattern known as Usual Interstitial Pneumonia (UIP).
  • Non-Specific Interstitial Pneumonia (NSIP): Often associated with connective tissue diseases but can be idiopathic. Generally has a better prognosis than IPF.
  • Cryptogenic Organising Pneumonia (COP).
  • Other rare forms.

3. Granulomatous ILD: (e.g., Sarcoidosis).

How would you investigate this patient?

Investigation aims to confirm the presence of ILD, identify the specific type, assess its severity, and detect complications.

  1. Initial Investigations:
    • Chest X-ray (CXR): May show reduced lung volumes and characteristic reticulo-nodular shadowing, predominantly in the lower zones.
    • Autoimmune Screen: Blood tests such as ANA, ENA, and Rheumatoid Factor to look for an underlying connective tissue disease.
  2. Key Diagnostic Investigations:
    • High-Resolution Computed Tomography (HRCT): This is the most important imaging tool. The pattern on HRCT is often diagnostic.
      • Typical UIP Pattern (for IPF): Characterized by basal and subpleural predominant reticulation, honeycombing, and traction bronchiectasis, with a lack of features suggesting an alternative diagnosis.
      • NSIP Pattern: Typically shows ground-glass opacities with less honeycombing.
    • Pulmonary Function Tests (PFTs): Will demonstrate a classic restrictive pattern (reduced FVC and FEV1 with a normal or high FEV1/FVC ratio) and reduced total lung capacity. The Diffusion Capacity (DLCO/KCO) will be significantly reduced, indicating impaired gas exchange.
  3. Assessing for Complications:
    • 2D Echocardiogram: Essential to assess for pulmonary hypertension and right ventricular dysfunction.
    • Six-Minute Walk Test: To objectively measure exercise capacity and oxygen desaturation.
  4. Further Investigations (if diagnosis is uncertain):
    • Bronchoscopy with Bronchoalveolar Lavage (BAL): Can help rule out infections and may support certain diagnoses (e.g., high lymphocytes in HP or NSIP).
    • Surgical Lung Biopsy: The ultimate gold standard if non-invasive tests are inconclusive. This is usually discussed in a multidisciplinary team (MDT) meeting.
How would you manage this patient?

Management depends heavily on the specific type of ILD and is best coordinated by a specialist respiratory MDT.

General and Supportive Management for all ILD patients:

  • Patient Education and Counselling: This is vital, especially for progressive diseases like IPF which have a poor prognosis.
  • Pulmonary Rehabilitation: Proven to improve symptoms of breathlessness and quality of life.
  • Ambulatory Oxygen Therapy: For patients who desaturate on exertion.
  • Long-Term Oxygen Therapy (LTOT): For those with resting hypoxemia.
  • Symptom Control: Treatment for cough and management of anxiety/depression.

Specific Management based on ILD type:

  • ILD due to CTD or HP: The primary treatment is immunosuppression to control the underlying inflammation, often with steroids (Prednisolone) and other agents like Mycophenolate Mofetil (MMF). Removing the trigger is crucial in HP.
  • Idiopathic Pulmonary Fibrosis (IPF):
    • Immunosuppression with steroids is not effective and can be harmful in IPF.
    • The standard of care is treatment with anti-fibrotic medications.
      • Pirfenidone and Nintedanib are two drugs that have been shown to slow the rate of decline in lung function in patients with mild to moderate IPF.
    • Lung Transplantation: Should be considered for suitable younger patients with advanced disease.
Source: NICE guideline [NG192] Idiopathic pulmonary fibrosis in adults: diagnosis and management (2022); BTS Clinical Statement on pulmonary fibrosis (2020).

Adult Medicine Long Case: A Broad Approach to Chest Pain

Patient Summary

A 60-year-old man presents to the emergency department with a 2-hour history of central chest pain. He describes the pain as “sharp” and notes it is worse when he takes a deep breath. His past medical history is significant for hypertension and he is a current smoker. On examination, he is anxious but alert. His blood pressure is 150/90 mmHg, heart rate is 95/min, respiratory rate is 22/min, and SpO2 is 97% on room air. His heart sounds are normal, and his chest is clear on auscultation.

How do you perform a thorough analysis of the symptom of chest pain?

A systematic and detailed history of the pain is the most important step in narrowing the differential diagnosis. The SOCRATES mnemonic is a useful framework.

  • Site: Where is the pain? Central, left-sided, retrosternal? Can the patient point to it with one finger (suggests musculoskeletal) or a whole fist (Levine’s sign for ischaemia)?
  • Onset: Was it sudden and instantaneous (e.g., aortic dissection, pneumothorax) or did it build up over minutes (e.g., ACS)?
  • Character: What does it feel like? Heavy, tight, crushing, pressure (ischaemia)? Sharp, stabbing (pleuritic, pericarditic)? Tearing, ripping (dissection)? Burning (reflux)?
  • Radiation: Does the pain travel anywhere? To the arm(s), jaw, neck (ischaemia)? Through to the back (dissection, pancreatitis)?
  • Associations: What other symptoms are present? Breathlessness, sweating, nausea (ACS, PE)? Fever, cough (pneumonia)? Dizziness, syncope (PE, dissection)?
  • Timing / Duration: Is it constant or does it come and go? How long does it last? (Angina lasts minutes, ACS lasts >20 mins).
  • Exacerbating & Relieving factors: What makes it worse or better? Worse on exertion, better with rest (angina)? Worse on deep inspiration or coughing (pleuritic pain)? Worse on lying flat, better sitting forward (pericarditis)? Relieved by antacids (reflux)?
  • Severity: Ask the patient to score the pain out of 10.
What are the life-threatening cardiac causes of chest pain and their key features?

The immediate priority is to rule out conditions that can kill. These are often called the “lethal six” in emergency medicine.

Condition Key History Features Key Examination Findings
Acute Coronary Syndrome (ACS) Central, crushing/heavy pain >20 mins. Radiation to arm/jaw. Associated with sweating, nausea, breathlessness. Risk factors (DM, HTN, smoking). May be normal. Can have signs of heart failure (S3, crepitations), new murmurs, or be in cardiogenic shock.
Aortic Dissection Sudden, instantaneous onset of severe “tearing” or “ripping” pain, typically radiating through to the interscapular area. Hypertension, unequal blood pressure between arms (>20mmHg difference), unequal pulses, new murmur of aortic regurgitation.
Pulmonary Embolism (PE) Sudden onset of pleuritic chest pain and/or breathlessness. May have hemoptysis, syncope. Risk factors (recent surgery, immobility, malignancy, DVT). Tachycardia, tachypnoea, low-grade fever. Signs of DVT in the legs. In massive PE, raised JVP and hypotension.
Tension Pneumothorax Sudden onset of pleuritic chest pain and severe breathlessness. Tracheal deviation away from the affected side, hyper-resonant percussion note, and absent breath sounds on the affected side. Hypotension.
Pericarditis with Tamponade Sharp, pleuritic, central pain, worse lying flat, better sitting forward. Pericardial friction rub. With tamponade, Beck’s triad: Hypotension, raised JVP, and muffled heart sounds.
Oesophageal Rupture (Boerhaave) Severe retrosternal pain following forceful vomiting or retching. Subcutaneous emphysema (crepitus in the neck), signs of sepsis.
How do you investigate a patient with undifferentiated acute chest pain?

The approach is rapid and systematic, aimed at diagnosing or excluding life-threatening conditions.

  1. Immediate Actions (within the first 10 minutes):
    • ABCDE Assessment: Secure airway, give oxygen only if SpO2 <94%.
    • IV Access and Bloods: Secure IV access. Send bloods for FBC, U&Es, coagulation screen, and serial Troponins.
    • 12-Lead ECG: This is the single most important initial test and must be performed and interpreted immediately. Look for ST elevation/depression, T-wave inversion, new LBBB, or signs of PE (S1Q3T3).
  2. Initial Imaging:
    • Chest X-ray (CXR): A portable CXR is essential. Look for a widened mediastinum (aortic dissection), pneumothorax, consolidation (pneumonia), or features of heart failure.
  3. Further Investigations based on suspected diagnosis:
    • Suspected ACS: Serial ECGs and troponins. Use a validated pathway (e.g., ESC 0/1-hour or 0/2-hour algorithm) to rule-in or rule-out MI quickly.
    • Suspected Aortic Dissection: CT Angiogram of the aorta is the definitive imaging test.
    • Suspected Pulmonary Embolism: Calculate a Wells’ score. If PE is likely, perform a CT Pulmonary Angiogram (CTPA). If unlikely, a D-dimer test can help rule it out.
    • Suspected Pericarditis: Echocardiogram to look for a pericardial effusion.
Source: 2020 ESC Guidelines for the management of acute coronary syndromes in patients presenting without persistent ST-segment elevation.

Adult Medicine Long Case: Chronic Cough & Hemoptysis (Tuberculosis)

Patient Summary

Mrs. X, a 45-year-old woman with an 8-year history of Type 2 Diabetes, presents with a chronic cough for 3 months. She has felt generally unwell with mild afternoon fevers. The cough is sometimes dry and sometimes productive. For the last few days, she has noticed blood in her sputum (hemoptysis). She denies any contact with tuberculosis but has lost 8 kg over the past 3 months. On examination, she appears wasted. There is no clubbing or lymphadenopathy. Respiratory examination reveals a trachea deviated to the right, reduced movements in the right apical area, which is also dull to percussion and has bronchial breathing on auscultation. These are classical signs of right upper lobe fibrosis and consolidation.

What is the most likely diagnosis and why do you say so?

The most likely diagnosis is Pulmonary Tuberculosis (TB).

The evidence is very strong based on the combination of symptoms, signs, and risk factors:

  • Cardinal Symptoms: The triad of chronic cough, hemoptysis, and significant weight loss is highly suggestive of TB.
  • Constitutional Symptoms: The history of evening/afternoon pyrexia (“evening fevers”) and general ill health are classic for a chronic infective process like TB.
  • Risk Factor: Her poorly controlled diabetes mellitus is a major risk factor for reactivation of latent TB.
  • Clinical Signs: The examination findings of right apical fibrosis (tracheal deviation, dullness, bronchial breathing in the upper zone) are pathognomonic for post-primary (reactivation) tuberculosis, which has a predilection for the lung apices.

What are the main differential diagnoses?

  • Bronchial Carcinoma (Lung Cancer): Can also present with cough, hemoptysis, and weight loss, especially in an older patient. However, the prominent evening fevers and classical apical signs make TB more likely.
  • Bronchiectasis: Can cause chronic productive cough and hemoptysis, but usually the history dates back to childhood infections, and the signs would be of coarse crepitations rather than apical fibrosis.
What investigations would you like to perform in this patient?

Investigations are to confirm the diagnosis, assess for drug resistance, and evaluate the extent of the disease.

  1. Microbiological Confirmation (The Gold Standard):
    • Sputum for Acid-Fast Bacilli (AFB): Three early morning sputum samples should be sent for Ziehl-Neelsen (ZN) staining to look for AFB.
    • Sputum for TB Culture: This is more sensitive than smear microscopy and is essential. Solid (LJ) or liquid (MGIT) cultures are used.
    • Molecular Testing (e.g., Xpert MTB/RIF): This is a rapid nucleic acid amplification test (NAAT) that should now be the initial test if available. It can detect M. tuberculosis DNA within hours and simultaneously checks for rifampicin resistance.
  2. Radiological Investigations:
    • Chest X-ray (CXR): Essential. Typical findings for post-primary TB are infiltrates, nodules, and cavitation, with a strong predilection for the upper lobes/apical segments. Evidence of fibrosis, volume loss, and calcification may also be seen.
  3. Other Investigations:
    • FBC: May show anaemia of chronic disease.
    • Inflammatory Markers: ESR and CRP will be elevated.
    • Tuberculin Skin Test (Mantoux) / IGRA: These tests indicate past infection/exposure but do not distinguish between latent and active TB. A strongly positive Mantoux test (>15mm induration) is suggestive of active disease, but false negatives can occur in malnourished or immunocompromised patients.
Source: WHO Consolidated Guidelines on Tuberculosis; NICE Guideline [NG33] Tuberculosis (2019).
Discuss the management of this patient.

The main objectives are to cure the patient, prevent transmission, and prevent the emergence of drug resistance.

1. General and Supportive Measures:

  • Patient Education: Counsel the patient on the disease, the importance of adherence to long-term treatment, and potential side effects.
  • Infection Control: In a hospital setting, the patient should be isolated in a negative pressure room if possible until they are smear-negative. Educate on cough etiquette (face mask, proper disposal of sputum).
  • Notification: TB is a notifiable disease. Public health authorities must be informed to enable contact tracing.
  • Directly Observed Therapy (DOTS): Referral to a DOTS program is standard to ensure adherence.

2. Anti-TB Drug Therapy:

Standard treatment for new, drug-sensitive pulmonary TB consists of a 6-month regimen divided into two phases:

  • Intensive Phase (2 months): A combination of four drugs taken daily: Isoniazid (H), Rifampicin (R), Pyrazinamide (Z), and Ethambutol (E).
  • Continuation Phase (4 months): A combination of two drugs taken daily: Isoniazid (H) and Rifampicin (R).

3. Monitoring of Treatment:

  • Baseline Tests: Perform baseline Liver Function Tests (LFTs) and a visual acuity/colour vision test (for Ethambutol) before starting treatment.
  • Sputum Smear Examination: Repeat sputum smear microscopy at the end of the intensive phase (2 months). If it is still positive, the intensive phase is extended for another month, and the sputum is sent for culture and drug susceptibility testing (DST). Further smears are checked at 5 and 6 months to confirm cure.
  • Monitoring for Adverse Effects: Educate the patient on the side effects of the drugs and to report any symptoms. Regular clinical follow-up is essential.
What are the common adverse effects of the anti-TB drugs?
Drug Common/Important Adverse Effects
Isoniazid (H) Hepatotoxicity (most important), peripheral neuropathy (prevented by co-administering Pyridoxine/Vitamin B6), hypersensitivity reactions.
Rifampicin (R) Hepatotoxicity (cholestatic), orange/red discoloration of body fluids (urine, tears – warn the patient!), flu-like symptoms, drug interactions (it is a potent enzyme inducer).
Pyrazinamide (Z) Hepatotoxicity, hyperuricemia (can precipitate gout), GI side effects, arthralgia.
Ethambutol (E) Optic neuritis (retrobulbar neuritis) – presents with reduced visual acuity and colour blindness (especially for red-green). This is dose-dependent and requires baseline and regular vision checks.

Adult Medicine Long Case: Generalized Oedema (Glomerular Disease)

Patient Summary (Nephrotic)

A 43-year-old female with a 3-year history of hypertension and hyperlipidaemia presents with generalized oedema for 1 week. This was associated with frothy urine for the past month. She also complains of symmetrical inflammatory arthritis affecting her MCP joints. Examination is significant for grade II hypertensive retinopathy, gross generalized oedema, and ascites.

Patient Summary (Nephritic)

A 24-year-old manual worker presents with passing red-coloured urine for 4 days. He has noticed a mild reduction in his urine output and swelling around his eyes in the morning. He has a history of an itchy, healing skin lesion on his left foot. On examination, his blood pressure is elevated at 160/100 mmHg, and he has mild periorbital oedema. A urine ward test is positive for 2+ proteins.

How do you differentiate between Nephrotic and Nephritic syndrome?

These are the two major clinical presentations of glomerular disease. The distinction is based on the degree of damage to the glomerulus.

Feature Nephrotic Syndrome Nephritic Syndrome
Pathophysiology Damage to podocytes leads to massive loss of protein. Inflammation of the glomeruli leads to damage of the entire glomerular filtration barrier.
Proteinuria Heavy proteinuria (>3.5g/24h), causing frothy urine. This is the defining feature. Mild to moderate proteinuria (<3.5g/24h).
Haematuria Usually absent or minimal. Gross or microscopic haematuria with red cell casts. This is the defining feature. Causes red/brown “cola-coloured” urine.
Oedema Severe, generalized oedema (anasarca) due to low serum albumin (hypoalbuminaemia). Mild oedema (e.g., periorbital) due to salt and water retention.
Blood Pressure Can be normal or elevated. Hypertension is a common and prominent feature.
Serum Albumin Low (<2.5 g/dl or 25 g/L). Normal or slightly reduced.
Other Features Hyperlipidaemia, hypercoagulable state. Often associated with oliguria and an acute kidney injury (AKI).
How would you confirm a diagnosis of Nephrotic Syndrome and assess the aetiology?

Confirmation of Diagnosis (The Tetrad):

  1. Heavy Proteinuria: Confirm with a urine protein:creatinine ratio (uPCR) or a 24-hour urine collection. (uPCR > 200 mg/mmol or >3.5g/24h).
  2. Hypoalbuminaemia: Serum albumin < 2.5 g/dL (25 g/L).
  3. Oedema.
  4. Hyperlipidaemia.

Assessing the Aetiology:

The next step is to determine if the nephrotic syndrome is primary (idiopathic) or secondary to another condition.

  • Diabetes Mellitus: The most common secondary cause in adults. Screen with FBS/HbA1c.
  • Autoimmune Disease: This is suggested in the case summary by the arthritis. Screen with ANA, anti-dsDNA, C3/C4 levels to look for Lupus Nephritis.
  • Infections: Screen for Hepatitis B, C, and HIV.
  • Neoplasms: An important cause in older adults. Solid tumours and haematological malignancies (lymphoma, myeloma) can present with nephrotic syndrome.
  • Drugs: NSAIDs are a classic cause of Minimal Change Disease in adults.

A renal biopsy is required in almost all cases of adult-onset nephrotic syndrome to determine the histological pattern (e.g., Minimal Change Disease, FSGS, Membranous Nephropathy), which is crucial for guiding management and prognosis.

What are the possible complications of nephrotic syndrome?

Complications can be related to the disease itself or its treatment.

Disease-Related Complications:

  • Thromboembolism: There is a hypercoagulable state due to urinary loss of anticoagulant proteins (like Antithrombin III) and increased hepatic synthesis of pro-coagulant factors. This leads to a high risk of DVT, PE, and especially Renal Vein Thrombosis.
  • Infection: Increased risk of serious bacterial infection (especially from encapsulated organisms like S. pneumoniae) due to loss of immunoglobulins (IgG) and complement components in the urine.
  • Acute Kidney Injury (AKI): Can occur due to hypovolemia.
  • Hyperlipidaemia: Long-term risk of accelerated atherosclerosis.

Treatment-Related Complications:

  • Side effects of steroids and other immunosuppressive drugs (e.g., increased infection risk, Cushing’s syndrome, diabetes).
How do you investigate and manage the patient with Acute Nephritic Syndrome?

Investigation:

  1. Confirm Haematuria and Proteinuria: Urine microscopy to confirm red blood cells and look for RBC casts (pathognomonic of glomerulonephritis). Quantify proteinuria with uPCR.
  2. Assess Renal Function: Check Urea & Electrolytes to look for AKI.
  3. Identify Aetiology:
    • Post-Streptococcal Glomerulonephritis (PSGN): This is suggested by the healing skin lesion. Check ASOT (for throat infection) or Anti-DNAse B titre (more specific for skin infection). Serum complement C3 will be low.
    • IgA Nephropathy: The most common cause worldwide. Often presents with synpharyngitic haematuria (haematuria occurring at the same time as a URTI). Serum complement is normal.
    • Small Vessel Vasculitis (e.g., ANCA-associated): Screen with an ANCA test.
    • Lupus Nephritis: Screen with ANA, anti-dsDNA. Complement C3/C4 will be low.

Management:

  1. Admit the patient: For monitoring and management.
  2. Manage Symptomatically (Supportive Care): This is the mainstay of treatment for PSGN.
    • Monitor Fluid Balance: Maintain a strict input/output chart. Fluid restriction is often necessary.
    • Manage Hypertension: Salt restriction and antihypertensive medications (often starting with a CCB or loop diuretic).
    • Diuretics: Use loop diuretics (e.g., Furosemide) to manage fluid overload.
  3. Specific Treatment: If the cause is an aggressive inflammatory condition like lupus nephritis or vasculitis, then specific immunosuppressive therapy (e.g., high-dose steroids, cyclophosphamide) is required, guided by a renal biopsy.
Source: KDIGO 2021 Clinical Practice Guideline for the Management of Glomerular Diseases.

Adult Medicine Long Case: Jaundice (Cirrhosis)

Patient Summary

A 36-year-old male was admitted with massive hematemesis (vomiting blood). He has a history of three similar episodes in the past year, with one episode followed by 6 days of drowsiness and disorientation. He does not consume alcohol. A year ago, he was investigated for darkening of his skin and yellowish eyes and was also diagnosed with diabetes mellitus. On examination, he is pale and icteric with generalized hyperpigmentation. He has several stigmata of chronic liver disease including gynecomastia, loss of body hair, palmar erythema, and testicular atrophy. Abdominal examination reveals splenomegaly (5cm below the costal margin) and gross ascites. The liver is not palpable.

What is your diagnosis and why do you say so?

The diagnosis is Decompensated Chronic Liver Disease (Cirrhosis) with Portal Hypertension.

This is based on evidence of three key components:

  1. Chronic Liver Disease:
    • Stigmata of CLD: The patient displays numerous peripheral signs of chronic liver failure, including icterus (jaundice), palmar erythema, gynecomastia, loss of body hair, and testicular atrophy.
  2. Portal Hypertension:
    • Variceal Bleeding: The presentation with massive hematemesis, in the context of CLD, is almost certainly due to bleeding from oesophageal varices, a direct consequence of high pressure in the portal venous system.
    • Splenomegaly and Ascites: These are classic signs of portal hypertension.
  3. Decompensation (Complications):
    • Variceal Bleed: This is a life-threatening decompensating event.
    • Ascites: The presence of fluid in the abdomen is a sign of decompensation.
    • Hepatic Encephalopathy: The history of drowsiness and disorientation following a previous bleed is a classic presentation of hepatic encephalopathy.
How would you investigate this patient to confirm the diagnosis and aetiology?

Investigations aim to confirm cirrhosis, assess its severity (using scores like Child-Pugh and MELD), and identify the underlying cause.

Investigations to Confirm Cirrhosis:

  • Laboratory Tests:
    • LFTs: May show low/normal transaminases (AST>ALT is classic in alcoholic liver disease but can be seen in advanced cirrhosis of any cause), elevated bilirubin, and low serum albumin.
    • Coagulation: A high PT/INR indicates impaired synthetic function of the liver and is a key prognostic marker.
    • FBC: May show anaemia (from bleeding/hypersplenism), thrombocytopenia (a very common and early sign of portal hypertension), or pancytopenia (from hypersplenism).
  • Imaging:
    • Abdominal Ultrasound with Doppler: Will show a small, nodular liver, splenomegaly, ascites, and can assess the patency and flow in the portal vein.
    • Transient Elastography (FibroScan): A non-invasive test that measures liver stiffness and is now widely used to diagnose and stage fibrosis/cirrhosis.

Investigations for the Aetiology (in this non-alcoholic patient):

  • Viral Hepatitis Serology: Hepatitis B (HBsAg, anti-HBc) and Hepatitis C (anti-HCV).
  • Autoimmune Markers: ANA, Anti-Smooth Muscle (ASM) antibodies for autoimmune hepatitis.
  • Metabolic Causes:
    • Iron Studies (Serum Ferritin, Transferrin Saturation): The combination of cirrhosis, diabetes, and skin hyperpigmentation (“bronze diabetes”) makes Hereditary Haemochromatosis a very likely diagnosis.
    • Serum Ceruloplasmin and Copper studies: To exclude Wilson’s Disease, an important differential in a young patient with liver disease.
Discuss the management of acute variceal bleeding.

Acute variceal bleeding is a medical emergency with high mortality. Management requires a multi-pronged approach focused on resuscitation, controlling bleeding, and preventing complications.

  1. Resuscitation (ABCDE):
    • Secure airway, especially in a patient with altered consciousness, to prevent aspiration.
    • Insert two wide-bore IV cannulae and start crystalloid infusion.
    • Restrictive Blood Transfusion Strategy: Transfuse packed red cells to maintain a target haemoglobin of 7-8 g/dL. Over-transfusion can increase portal pressure and worsen bleeding.
    • Correct coagulopathy with FFP and platelets if indicated.
  2. Pharmacological Therapy:
    • Vasoactive Drugs: Start a vasoactive drug immediately to reduce portal pressure. Terlipressin (IV bolus) is the drug of choice. Octreotide is an alternative.
  3. Prophylactic Antibiotics:
    • All patients with a variceal bleed should receive prophylactic IV antibiotics (e.g., Ceftriaxone) as bacterial infection is a major trigger and complication, increasing the risk of re-bleeding and death.
  4. Urgent Endoscopy:
    • Arrange an urgent upper GI endoscopy (within 12 hours).
    • The therapeutic procedure of choice is endoscopic variceal band ligation.
  5. If Endoscopy Fails:
    • If bleeding cannot be controlled, options include balloon tamponade (Sengstaken-Blakemore tube) as a temporary bridge to definitive therapy.
    • The definitive salvage therapy is a Transjugular Intrahepatic Portosystemic Shunt (TIPSS).
Source: European Association for the Study of the Liver (EASL) Clinical Practice Guidelines on portal hypertension (2022).
How do you manage ascites and hepatic encephalopathy?

Management of Ascites:

  • Dietary Salt Restriction: Daily sodium intake should be less than 2g/day.
  • Diuretic Therapy: The mainstay is a combination of an aldosterone antagonist and a loop diuretic.
    • Start with oral Spironolactone (e.g., 100mg daily).
    • Add oral Furosemide (e.g., 40mg daily).
    • Titrate doses based on weight loss and electrolytes, maintaining a 100:40 ratio of spironolactone to furosemide.
  • Diagnostic Paracentesis: A peritoneal tap should be done at first presentation to calculate the SAAG (Serum-Ascites Albumin Gradient) to confirm it is due to portal hypertension, and to check the white cell count to rule out Spontaneous Bacterial Peritonitis (SBP).
  • Large Volume Paracentesis (LVP): For tense ascites, LVP can be performed for symptomatic relief. Human Albumin Solution (HAS) should be infused (8g per litre of fluid removed) if more than 5 litres are drained to prevent post-paracentesis circulatory dysfunction.

Management of Hepatic Encephalopathy (HE):

  • Identify and Treat the Precipitating Cause: This is the most important step. Common triggers include infection (e.g., SBP), GI bleeding, electrolyte disturbance (especially hypokalaemia from diuretics), constipation, and sedating drugs.
  • Specific Therapy (to reduce ammonia levels):
    • Lactulose: A non-absorbable disaccharide given orally to aim for 2-3 soft stools per day. It works by acidifying the colon, converting ammonia (NH3) to the non-absorbable ammonium (NH4+).
    • Rifaximin: A non-absorbable antibiotic that reduces ammonia-producing gut bacteria. It is used as an add-on to lactulose for secondary prevention of HE.

Adult Medicine Long Case: Diabetes Mellitus with Complications

Patient Summary

A 65-year-old man with a 4-year history of Type 2 Diabetes Mellitus (T2DM) presents with a 3-day history of painful, localized swelling of both upper legs, associated with high-grade fever and chills. He is known to have macrovascular and microvascular complications, including ischaemic heart disease (awaiting CABG) and a painful symmetric diabetic neuropathy. He is poorly compliant with his medication (metformin, gliclazide, atorvastatin, losartan, aspirin) and has poor knowledge of his disease. On examination, he has bilateral cellulitis of the lower limbs and evidence of peripheral neuropathy. His BP is high at 190/80 mmHg, and fundoscopy shows non-proliferative diabetic retinopathy.

How would you optimize the management of this patient?

The management is two-fold: treating the acute presenting problem (cellulitis) and then creating a long-term, holistic plan to optimize his diabetes and cardiovascular risk management.

Acute Management:

  1. Treat the bilateral lower limb cellulitis with appropriate IV antibiotics.
  2. Control his severe hyperglycemia, which may require a variable rate insulin infusion initially.
  3. Manage his severe hypertension.

Long-Term Optimization (The First Step is a Comprehensive Review):

  1. Review the Diagnosis: Confirm the diagnosis of T2DM based on standard criteria (HbA1c, fasting/random glucose).
  2. Patient Education: This is the cornerstone. The patient is poorly compliant and has poor knowledge. He needs structured education on diet, lifestyle, medication, self-monitoring of blood glucose, and foot care.
  3. Optimize Glycaemic Control: Review his current medication and set a personalized HbA1c target.
  4. Manage Comorbidities and Risk Factors: Aggressively manage blood pressure and lipids.
  5. Screen for and Manage Complications: Regular screening for retinopathy, nephropathy, and neuropathy.
How do you approach the pharmacological management and optimizing therapy for T2DM?

The modern approach to T2DM therapy is patient-centered, moving beyond just glucose-lowering to a strategy that also reduces cardiovascular and renal risk.

1. Initial Therapy:

  • Metformin remains the first-line drug for T2DM, along with comprehensive lifestyle modification, unless contraindicated.

2. Subsequent Therapy (The Modern Approach):

  • The choice of the second agent is now primarily driven by the presence of comorbidities.
  • For a patient with established atherosclerotic cardiovascular disease (ASCVD) like this man (IHD), or indicators of high risk:
    • The recommendation is to add an agent with proven cardiovascular benefit, independent of the HbA1c level.
    • The preferred choices are either a GLP-1 receptor agonist (e.g., Liraglutide, Semaglutide) OR an SGLT2 inhibitor (e.g., Empagliflozin, Dapagliflozin).
  • For a patient with heart failure or chronic kidney disease (CKD):
    • An SGLT2 inhibitor is the recommended agent due to its proven benefits in reducing progression of both conditions.

For this Patient: He is on metformin and gliclazide (a sulphonylurea). Given his IHD, he is a prime candidate for a change in therapy. One would strongly consider replacing the gliclazide with an SGLT2 inhibitor or a GLP-1 RA to provide cardiovascular protection in addition to glucose lowering.

Source: ADA/EASD Consensus Report: Management of Hyperglycemia in Type 2 Diabetes (2022).
How do you manage cardiovascular complications and risk factors in this patient?

This is a critical aspect of management, as cardiovascular disease is the leading cause of death in patients with diabetes.

1. Hypertension Management:

  • Target: A blood pressure target of < 130/80 mmHg is recommended for patients with diabetes.
  • Medication: An ACE inhibitor or an ARB (he is on losartan) is the first-line choice, especially in patients with evidence of nephropathy. Often, combination therapy with a CCB or diuretic is required.

2. Lipid Management:

  • Statin Therapy: This patient, with T2DM and established ASCVD, is in the highest risk category. He must be on a high-intensity statin therapy (e.g., Atorvastatin 80mg) to achieve an LDL-C target of < 1.4 mmol/L (<55 mg/dL).
  • If the LDL target is not met on a maximal statin dose, consider adding Ezetimibe.

3. Antiplatelet Therapy:

  • He is already on low-dose aspirin for secondary prevention, which is appropriate.
How do you discuss and manage the microvascular complications in this patient?

1. Diabetic Retinopathy:

  • He has non-proliferative retinopathy. The key is to prevent progression.
  • Management: Intensive glycaemic control and optimal blood pressure control are the most effective interventions.
  • Follow-up: He needs regular (annual) digital retinal photography screening to detect progression to proliferative disease or the development of maculopathy, which would require urgent referral to an ophthalmologist for laser photocoagulation or intravitreal anti-VEGF injections.

2. Diabetic Nephropathy:

  • Screening: This is done annually with a urine albumin:creatinine ratio (uACR) and an estimated GFR (eGFR).
  • Management: If microalbuminuria is present, an ACE inhibitor or ARB should be started (he is on losartan) as they are reno-protective. As noted above, SGLT2 inhibitors are also now strongly recommended for their renal benefits. Optimizing BP and glycaemic control is also crucial.

3. Diabetic Neuropathy:

  • He has painful peripheral neuropathy.
  • Management of Pain:
    • First-line pharmacological agents include: Amitriptyline, Duloxetine, Gabapentin, or Pregabalin.
  • Foot Care: This is paramount. He has sensory loss, putting him at high risk for foot ulcers. He needs structured education on:
    • Daily foot inspection.
    • Proper footwear (avoiding walking barefoot).
    • Nail care.
    • Urgent medical attention for any new cut, blister, or swelling.
    • Regular review by a podiatrist.

Adult Medicine Long Case: Rheumatoid Arthritis

Patient Summary

A 43-year-old widow and mother of three presents with a 5-week history of pain and swelling in the joints of her hands and wrists. The pain is symmetrical, affecting the metacarpophalangeal (MCP) and proximal interphalangeal (PIP) joints. She experiences significant morning stiffness lasting for more than an hour. She also has pain in both shoulders and knees. She complains of fatigue and symptoms of anaemia but denies any skin rashes or oral ulcers. Her social support is poor, and her functional status is declining. On examination, she is pale. There is evidence of symmetrical synovitis (swelling, warmth, tenderness) of the MCP and PIP joints, with early deformities. Examination of the nervous system reveals a sensory polyneuropathy.

What is your diagnosis and why do you say so?

The diagnosis is Rheumatoid Arthritis (RA).

The diagnosis is based on the classic features of an inflammatory polyarthritis:

  • Pattern of Joint Involvement: The arthritis is symmetrical and affects the small joints of the hands (MCPs, PIPs) and wrists, as well as large joints (shoulders, knees). Crucially, the distal interphalalangeal (DIP) joints are spared, which is typical for RA.
  • Inflammatory Nature: This is confirmed by the prolonged morning stiffness lasting more than 1 hour and the examination findings of synovitis (joint swelling and warmth).
  • Systemic Features: Her symptoms of fatigue, anaemia, and the presence of a peripheral neuropathy are consistent with the systemic, extra-articular nature of RA.
  • Duration: The symptoms have lasted for 5 weeks, consistent with an early presentation.

Differential Diagnosis (D/D):

  • Systemic Lupus Erythematosus (SLE): SLE can cause a similar pattern of arthritis, but it is typically non-erosive and would usually be accompanied by other features like skin rashes (malar rash), oral ulcers, or alopecia, which this patient denies.
  • Spondyloarthropathies (e.g., Psoriatic Arthritis): Less likely due to the symmetrical pattern and lack of axial (spine) involvement or other features like psoriasis.
  • Generalized Osteoarthritis (OA): This is a non-inflammatory process, with morning stiffness usually lasting less than 30 minutes. It typically involves the DIP joints.
How would you establish a diagnosis and investigate this patient?

The diagnosis is primarily clinical but is supported by serological and radiological investigations.

1. Diagnostic Criteria:

  • The 2010 ACR/EULAR Classification Criteria for RA can be used. A score of ≥6/10 confirms the diagnosis. This patient would score highly based on the number and type of joints involved (small joints), duration (>6 weeks is a criterion, she is at 5 weeks), and likely elevated inflammatory markers.

2. Investigations to Establish the Diagnosis:

  • Inflammatory Markers: ESR and CRP will be elevated, confirming the inflammatory nature of the arthritis.
  • Serology (Autoantibodies):
    • Rheumatoid Factor (RF): Will be positive in ~70-80% of patients.
    • Anti-Cyclic Citrullinated Peptide (Anti-CCP) antibodies: This is a more specific marker for RA than RF. A positive Anti-CCP is highly predictive of RA and is associated with more aggressive, erosive disease.
  • Imaging:
    • X-rays of the hands and feet: In early disease, X-rays may only show soft tissue swelling and periarticular osteopenia. The classical changes of bony erosions and joint space narrowing occur later.

3. Baseline Investigations (Before starting treatment):

  • FBC (to check for anaemia), Liver Function Tests (LFTs), and Renal Function Tests (U&Es) are essential before starting DMARD therapy.
  • Screening for latent TB and viral hepatitis (Hep B/C) is also required before commencing certain biologic therapies.
How will you treat this patient?

The management of RA is guided by a “Treat to Target” strategy, aiming for clinical remission or low disease activity to prevent joint damage and disability. Treatment should be initiated by a rheumatologist as early as possible.

1. General Management:

  • Patient Education: Counsel the patient about the chronic nature of RA, the importance of adherence, and the goals of treatment.
  • Multidisciplinary Team (MDT) Input: Refer to physiotherapy (for exercise programs to maintain joint function and muscle strength) and occupational therapy (for joint protection advice, splints, and aids for daily living).
  • Cardiovascular Risk Management: RA is an independent risk factor for cardiovascular disease. It is crucial to manage other risk factors like smoking, hypertension, and lipids aggressively.

2. Pharmacological Management:

  • Disease-Modifying Anti-Rheumatic Drugs (DMARDs):
    • This is the mainstay of treatment and should be started as soon as the diagnosis is made.
    • Methotrexate (MTX) is the first-line anchor drug. It is given once weekly. Folic acid should be co-prescribed to reduce side effects.
    • If MTX is contraindicated or not tolerated, other conventional synthetic DMARDs (csDMARDs) like Leflunomide or Sulfasalazine can be used.
  • Bridging Therapy with Corticosteroids:
    • DMARDs take several weeks to work. A short course of oral steroids (e.g., Prednisolone) or an intramuscular steroid injection (e.g., Depo-Medrone) is used as a “bridge” to quickly control inflammation and provide symptom relief while waiting for the DMARD to take effect.
  • Step-Up Therapy (if target is not reached):
    • If the disease is not controlled after 3-6 months on monotherapy, the strategy is to step up to combination therapy (e.g., MTX + Sulfasalazine + Hydroxychloroquine) or to add a biologic DMARD (bDMARD) or a targeted synthetic DMARD (tsDMARD).
Source: EULAR recommendations for the management of rheumatoid arthritis (2022); NICE Guideline [NG100] Rheumatoid arthritis in adults: management (2020).
How would you monitor a patient on Methotrexate (MTX)?

Monitoring is crucial due to the potential for significant toxicity.

  1. Patient Instruction: Emphasize that it is taken ONCE WEEKLY. Daily dosing is a major cause of fatal toxicity. Provide clear instructions on the dose and day of administration.
  2. Baseline Bloods: FBC, U&Es, and LFTs before starting.
  3. Regular Blood Monitoring:
    • Check FBC, U&Es, and LFTs every 2 weeks for the first 6 weeks, then monthly for 3 months, and then at least every 12 weeks thereafter.
    • The goal is to detect bone marrow suppression (leukopenia, thrombocytopenia) and hepatotoxicity.
  4. Monitoring for Other Side Effects:
    • MTX-induced Lung Disease (Pneumonitis): Warn the patient to report any new onset of dry cough or shortness of breath. This is an uncommon but serious complication.
    • Gastrointestinal Effects: Nausea and oral ulcers are common. Folic acid helps to reduce these.
  5. Contraception Advice: MTX is teratogenic. Advise both male and female patients of reproductive age to use effective contraception during treatment and for at least 6 months after stopping.

Adult Medicine Long Case: Backache

Patient Summary

A 30-year-old man presents with a 4-month history of lower back pain. He describes the pain as a dull ache, which is worst in the morning upon waking, associated with significant stiffness that lasts for about an hour. He finds that the pain improves with exercise and activity during the day but is not relieved by rest. The pain sometimes awakens him in the second half of the night and alternates between his left and right buttocks. He has also had a painful, swollen right knee for the past month. He denies any history of trauma. On examination, he has reduced range of motion in his lumbar spine, particularly in forward flexion (Modified Schober test is abnormal). There is tenderness over the sacroiliac joints and evidence of synovitis in the right knee.

How do you differentiate between inflammatory and mechanical back pain?

This is the most critical first step in evaluating a patient with backache. The history is key.

Feature Inflammatory Back Pain (e.g., Spondyloarthropathy) Mechanical Back Pain (e.g., Degenerative Disc Disease)
Age of Onset Typically < 40 years Can occur at any age, more common with increasing age
Onset Insidious (gradual) Often acute, may relate to an injury
Morning Stiffness Present, prolonged (> 30 minutes) Present, but brief (< 30 minutes)
Effect of Exercise Improves with exercise and activity Worsens with exercise and activity
Effect of Rest Not relieved by rest, may worsen Improves with rest
Night Pain Common, especially in the second half of the night, waking the patient Uncommon, unless pain is severe
Alternating Buttock Pain A very characteristic feature Not typical

This patient’s history is classical for Inflammatory Back Pain.

What are the key features of Spondyloarthropathy?

Spondyloarthropathy (SpA), now often termed Spondyloarthritis, is a group of inflammatory rheumatic diseases with shared features. The key features are:

  • Axial Involvement:
    • Sacroiliitis: Inflammation of the sacroiliac joints, causing buttock pain.
    • Spondylitis: Inflammation of the spine, causing inflammatory back pain and stiffness.
  • Peripheral Asymmetric Oligoarthritis:
    • Inflammation of a few joints (oligo-), typically in an asymmetric pattern, and predominantly affecting the large joints of the lower limbs (hips, knees, ankles).
  • Enthesitis:
    • Inflammation at the site of insertion of tendons and ligaments into bone. This is a hallmark feature of SpA. Common sites include the Achilles tendon insertion and the plantar fascia insertion, causing heel pain.
  • Dactylitis (“Sausage Digit”):
    • Uniform, painful swelling of an entire finger or toe.
  • Extra-articular Manifestations:
    • Anterior Uveitis: The most common extra-articular feature. Presents with a painful, red eye and photophobia.
    • Psoriasis: Skin and nail changes.
    • Inflammatory Bowel Disease (IBD): Crohn’s disease or Ulcerative Colitis.
    • Cardiac involvement (e.g., aortitis, conduction blocks) can occur.
  • Genetic Association: Strong association with the HLA-B27 gene.
  • Good Response to NSAIDs.

This patient has axial involvement, peripheral arthritis, and likely enthesitis, making a diagnosis of SpA (likely Ankylosing Spondylitis) highly probable.

When assessing back pain, what are the “Red Flag” symptoms?

“Red Flags” are alarm symptoms that suggest a serious underlying spinal pathology (e.g., malignancy, infection, fracture, or cauda equina syndrome) that requires urgent investigation.

  • Age of onset < 20 years or > 50 years.
  • History of Cancer.
  • Constitutional Symptoms: Unexplained weight loss, fever, night sweats.
  • Recent bacterial infection or IV drug use.
  • Immunosuppression.
  • Pain Characteristics:
    • Thoracic pain (less common for mechanical pain, more sinister).
    • Nocturnal pain that is progressive and disturbs sleep.
    • Pain unresponsive to simple analgesics and rest.
  • Neurological Symptoms (Cauda Equina Syndrome – A Neurosurgical Emergency):
    • Bilateral sciatica or leg weakness.
    • Saddle anaesthesia (numbness in the perineal region).
    • Bladder or bowel dysfunction (urinary retention or incontinence, faecal incontinence).
  • History of significant trauma.
Source: NICE Guideline [NG59] Low back pain and sciatica in over 16s: assessment and management (2020).
How would you investigate and manage this patient with suspected Spondyloarthropathy?

Investigations:

  1. Baseline Bloods: FBC, U&Es, LFTs.
  2. Inflammatory Markers: ESR and CRP are often, but not always, elevated.
  3. Genetic Marker: Test for HLA-B27. While not diagnostic (it can be present in the healthy population), it is positive in >90% of patients with Ankylosing Spondylitis and supports the diagnosis.
  4. Imaging:
    • X-ray of the sacroiliac joints and spine: May show characteristic changes like sacroiliitis (sclerosis, erosions, joint space fusion), syndesmophytes, and squaring of vertebral bodies, leading to a “bamboo spine” appearance in advanced disease. X-ray changes can take years to develop.
    • MRI of the sacroiliac joints: This is the most sensitive imaging modality for detecting early disease. It can show active inflammation (bone marrow oedema) long before changes are visible on an X-ray.

Management:

Management is multidisciplinary, with the goals of controlling pain and stiffness, preventing structural damage, and maintaining function.

  1. Non-Pharmacological:
    • Physiotherapy: This is the cornerstone of management. A regular, lifelong exercise program focusing on spinal mobility and posture is essential.
    • Patient Education.
  2. Pharmacological:
    • NSAIDs (Non-Steroidal Anti-Inflammatory Drugs): This is the first-line drug treatment. They should be taken on a regular basis at maximum tolerated dose to control symptoms.
    • DMARDs (Disease-Modifying Anti-Rheumatic Drugs): Conventional DMARDs like Sulfasalazine are effective for peripheral arthritis but not for axial disease.
    • Biologic Therapies: For patients with active axial disease that has not responded to NSAIDs, biologic DMARDs are the treatment of choice. TNF-inhibitors (e.g., Adalimumab, Etanercept) and IL-17 inhibitors (e.g., Secukinumab) are highly effective.

Adult Medicine Long Case: Systemic Lupus Erythematosus (SLE)

Patient Summary

A 25-year-old female presents with a 2-week history of progressive bilateral ankle swelling, reduced urine output, and difficulty in breathing. She also complains of low-grade fever and pain in multiple joints (polyarthralgia). Over the last 2 days, she has developed oral ulcers and a facial rash. On examination, she has a classic malar (“butterfly”) rash across her cheeks and nose, oral ulcers on the hard palate, and significant bilateral lower limb oedema.

How would you establish a diagnosis of SLE?

SLE is a multisystem autoimmune disease with a wide range of clinical presentations. The diagnosis is made based on a combination of characteristic clinical features and the presence of specific autoantibodies, using classification criteria.

Classification Criteria:

  • The 2019 EULAR/ACR classification criteria are now standard. An ANA (Antinuclear Antibody) is an entry criterion; a patient must have a positive ANA (titer ≥ 1:80) to be considered for classification.
  • After a positive ANA, a scoring system is applied based on weighted clinical and immunological criteria from seven domains (constitutional, haematologic, neuropsychiatric, mucocutaneous, serosal, musculoskeletal, renal).
  • A score of ≥10 confirms the diagnosis.

This patient’s presentation is highly suggestive of SLE with active lupus nephritis:

  • Clinical criteria present: Low-grade fever, polyarthralgia, oral ulcers, acute cutaneous lupus (malar rash), and likely renal involvement (oedema, reduced urine output).
  • Immunological workup is essential: You would request ANA, anti-dsDNA, anti-Sm, complement levels (C3, C4), and antiphospholipid antibodies.

A diagnosis can also be made in a patient with biopsy-proven lupus nephritis in the presence of a positive ANA or anti-dsDNA.

Source: Aringer M, et al. 2019 EULAR/ACR classification criteria for SLE. Ann Rheum Dis. 2019.
How would you manage lupus nephritis?

The management of lupus nephritis is the most important factor in determining the patient’s long-term prognosis. This patient’s presentation with oedema and reduced urine output is a medical emergency.

  1. Confirmation and Classification (Renal Biopsy):
    • A renal biopsy is mandatory in any patient with clinical or laboratory evidence of active nephritis.
    • It is crucial for confirming the diagnosis and, most importantly, for histological classification (ISN/RPS classification Classes I-VI). The class of nephritis determines the intensity of treatment. Proliferative forms (Class III and IV) are the most aggressive and require immediate immunosuppression.
  2. General Management:
    • Strict blood pressure control (target <130/80 mmHg), usually with an ACE inhibitor or ARB for their antiproteinuric effects.
    • Management of oedema with diuretics.
  3. Immunosuppressive Therapy for Proliferative Nephritis (Class III/IV):
    • Treatment is divided into two phases:
      1. Induction Therapy (to achieve remission): This is an intensive phase lasting 3-6 months. The standard approach is high-dose corticosteroids (often starting with IV methylprednisolone pulses) combined with another potent immunosuppressant. The choice is typically between:
        • Mycophenolate Mofetil (MMF): Now often preferred due to a better side effect profile, especially regarding fertility.
        • Cyclophosphamide (CPP): A potent alkylating agent, highly effective but with significant toxicity.
      2. Maintenance Therapy (to prevent relapse): After achieving remission, the patient is switched to a less intensive maintenance regimen, typically with MMF or Azathioprine, which is continued for several years.
  4. Background Therapy for all SLE patients:
    • Hydroxychloroquine (HCQ) should be commenced in all patients with SLE unless contraindicated. It has been shown to reduce disease flares, improve survival, and has antithrombotic and lipid-lowering effects.
What are the problems associated with Cyclophosphamide (CPP) therapy?

Cyclophosphamide is a highly effective but toxic medication. The main problems are:

  • Infertility / Premature Ovarian Failure: This is a major concern, especially in young female patients. The risk is related to the cumulative dose. Patients must be counselled regarding this risk before starting, and fertility preservation options (e.g., sperm/ovum banking, GnRH agonist treatment during therapy) should be offered.
  • Hemorrhagic Cystitis: A toxic metabolite of CPP, acrolein, irritates the bladder lining, causing bleeding. This is prevented by ensuring adequate hydration and co-administering a protective agent called Mesna.
  • Bone Marrow Suppression: Can cause leukopenia and increase the risk of infection.
  • Increased Risk of Malignancy: Long-term risk of bladder cancer and haematological malignancies.
  • Opportunistic Infections.
What are the principles of planned pregnancy in a patient with SLE?

Pregnancy in SLE is high-risk and requires careful planning and multidisciplinary care involving the rheumatologist and obstetrician.

The General Principles are:

  1. Plan the Pregnancy: It must be planned. Unplanned pregnancies are associated with much higher risks.
  2. Achieve Disease Remission: The patient should be in a state of stable disease remission on pregnancy-compatible medications for at least 6 months before attempting conception. Active disease at conception dramatically increases the risk of maternal flares and adverse fetal outcomes.
  3. Adjust Medications:
    • Contraindicated drugs must be stopped well before conception: Mycophenolate (MMF), Cyclophosphamide, Methotrexate, ACE inhibitors.
    • Pregnancy-compatible drugs: Hydroxychloroquine (should be continued throughout pregnancy), Azathioprine, low-dose Prednisolone, and Tacrolimus can be used if necessary.
  4. Screen for High-Risk Antibodies:
    • Anti-Ro and Anti-La antibodies: If present, there is a risk of the baby developing neonatal lupus, including congenital heart block. The fetus requires regular monitoring with fetal echocardiograms.
    • Antiphospholipid antibodies (APLS): If present, the patient is at very high risk of thrombosis and pregnancy loss and requires treatment with low-dose aspirin and heparin throughout the pregnancy.

Adult Medicine Long Case: Anaemia

Patient Summary (Hemolytic Anaemia)

A 23-year-old girl presents with progressive shortness of breath, lethargy, and a gradual yellowish discoloration of her eyes and skin. She has a history of heavy menstrual bleeds. Three weeks ago, she had a sore throat with fever requiring hospital admission. On examination, she is lying comfortably but is markedly pale with mild jaundice. She has palpable cervical, axillary, and inguinal lymphadenopathy. A systolic ejection murmur is heard. There is no hepatosplenomegaly.

How do you evaluate anaemia in a patient? (A Prototypic Approach)

The evaluation of anaemia is a systematic process guided by the history, examination, and initial blood tests.

  1. History and Examination:
    • History: Look for clues to the cause. Ask about bleeding (malaena, menorrhagia), diet (veganism for B12), chronic disease (CKD, rheumatological), and symptoms of haemolysis (jaundice).
    • Examination: Look for signs of iron deficiency (koilonychia, angular stomatitis), B12 deficiency (neuropathy), or haemolysis (jaundice, splenomegaly).
  2. Review the Full Blood Count (FBC) and Classify:
    • The first step is to classify the anaemia based on the Mean Corpuscular Volume (MCV).
    • Microcytic (MCV < 80 fL): Small red cells.
    • Normocytic (MCV 80-100 fL): Normal-sized red cells.
    • Macrocytic (MCV > 100 fL): Large red cells.
  3. Further Investigations based on Morphological Type.
What are the causes of the different morphological forms of anaemia?
Microcytic (MCV < 80) Normocytic (MCV 80-100) Macrocytic (MCV > 100)
Iron Deficiency Anaemia (Most common) Anaemia of Chronic Disease (ACD) Megaloblastic (B12 / Folate deficiency)
Thalassaemia Trait Acute Blood Loss Alcohol / Liver Disease
Anaemia of Chronic Disease (can also be normocytic) Chronic Kidney Disease (CKD) Myelodysplastic Syndromes
Sideroblastic Anaemia Haemolytic Anaemia Reticulocytosis (high reticulocyte count)
Lead Poisoning Bone Marrow Failure (Aplastic Anaemia) Hypothyroidism
Why did you think of a hemolytic anaemia in this patient?

The diagnosis of hemolytic anaemia is strongly suggested by the combination of clinical features in this patient’s history and examination.

  • The Classic Triad of Extravascular Haemolysis:
    1. Anaemia: She is clinically pale and has symptoms of anaemia (shortness of breath, lethargy).
    2. Jaundice: She has yellowish discoloration of her eyes and skin, caused by the breakdown of haemoglobin into unconjugated bilirubin.
    3. Splenomegaly: (Not present in this case, but a classic feature). The spleen is the primary site of removal of damaged red cells in extravascular haemolysis.
  • Precipitating Event: The haemolysis appears to have been triggered by a recent infection (sore throat), which is a common trigger for autoimmune haemolytic anaemia (AIHA).
  • Evidence of a Possible Underlying Cause: The presence of widespread lymphadenopathy in a young person with AIHA is a red flag for an underlying secondary cause, such as a lymphoproliferative disorder (lymphoma) or an autoimmune disease like SLE.
How will you investigate and manage this patient with suspected Autoimmune Hemolytic Anaemia (AIHA)?

Initial Investigations to Confirm Haemolysis:

  • FBC and Blood Picture: Typically shows a normocytic anaemia. The blood picture may show spherocytes and polychromasia (indicating reticulocytosis).
  • Reticulocyte Count: Will be high, indicating a healthy bone marrow trying to compensate for the peripheral destruction of red cells.
  • Markers of Red Cell Breakdown:
    • Serum Bilirubin: High unconjugated bilirubin.
    • Lactate Dehydrogenase (LDH): Will be high.
    • Haptoglobin: Will be low or absent (as it gets used up binding free haemoglobin).

Further Investigations to Identify the Cause (AIHA):

  • Direct Coombs Test (DCT) / Direct Antiglobulin Test (DAT): This is the key diagnostic test for AIHA. It detects the presence of antibodies and/or complement on the surface of the patient’s red blood cells. A positive test confirms the autoimmune nature of the haemolysis.

Management (Warm AIHA, often secondary to SLE/Lymphoma):

  1. Ensure Stability: Assess the patient’s haemodynamic status.
  2. Folic Acid: Start folic acid supplementation (5mg/d) as active haemolysis consumes folate stores.
  3. Immunosuppression:
    • Corticosteroids are the first-line treatment. High-dose oral Prednisolone (1mg/kg) is started.
  4. Blood Transfusion: Transfusion in AIHA is high-risk due to the autoantibodies making cross-matching difficult. It should be reserved only for patients who are severely symptomatic or haemodynamically unstable. If needed, transfuse slowly with the least incompatible blood available.
  5. Second-Line Therapy: If there is no response to steroids, second-line options include:
    • Rituximab (a monoclonal antibody against B-cells) is now often preferred.
    • Splenectomy can be considered.
    • Other immunosuppressants like Azathioprine or Mycophenolate.

Adult Medicine Long Case: Lower Limb Weakness (GBS)

Patient Summary

A 25-year-old, previously healthy woman, presents with progressively worsening weakness that started in her feet and has now ascended to involve her upper limbs. She also complains of difficulty in breathing. She recalls having an episode of gastroenteritis about 3 weeks ago. Her neurological examination is significant for symmetrical, flaccid weakness in both her lower and upper limbs, with a complete absence of reflexes (areflexia). Her sensory system is intact.

What is the most likely diagnosis?

The most likely diagnosis is Guillain-Barré Syndrome (GBS), specifically the most common subtype, Acute Inflammatory Demyelinating Polyneuropathy (AIDP).

This is based on the classic triad of features:

  1. Progressive, Symmetrical Weakness: The weakness is ascending, starting in the lower limbs and progressing upwards, which is the hallmark of GBS.
  2. Areflexia: The universal absence of deep tendon reflexes is a cardinal sign.
  3. Preceding Infection: There is a history of a preceding infectious illness (gastroenteritis, most commonly due to Campylobacter jejuni) about 1-4 weeks prior to the onset of weakness. This triggers the autoimmune response.

The absence of significant sensory loss is also characteristic of the classic AIDP variant.

How do you differentiate between an Upper Motor Neuron (UMN) and Lower Motor Neuron (LMN) pattern of weakness?

This is a fundamental concept in neurology. The patient with GBS presents with a classic LMN pattern.

Sign Upper Motor Neuron (UMN) Lesion Lower Motor Neuron (LMN) Lesion
Tone Increased (spasticity, “clasp-knife”) Reduced or absent (flaccid)
Reflexes Increased (hyperreflexia), clonus Diminished or absent (areflexia)
Plantar Response Extensor (upgoing toe – Babinski sign) Flexor (downgoing toe) or absent
Wasting Minimal (disuse atrophy) Pronounced, proportional to weakness
Fasciculations Absent May be present
How are you going to manage this patient?

GBS is a medical emergency due to the risk of respiratory failure and autonomic instability. Management requires hospitalization, ideally in an ICU or a high-dependency setting.

1. Monitoring (The Most Important Aspect):

  • Respiratory Monitoring: This is critical. The most important aspect is the regular assessment of respiratory capacity to detect impending respiratory failure.
    • Bedside Tests: Single breath counting (ability to count to 20 indicates adequate vital capacity) and checking for neck flexion weakness.
    • Objective Measurement: Serial measurement of Forced Vital Capacity (FVC) and negative inspiratory force (NIF). An FVC of less than 20 ml/kg is an indication for elective intubation and ventilation.
  • Autonomic and Cardiac Monitoring: Patients can have severe autonomic instability. Continuous ECG and frequent BP monitoring are essential to detect labile blood pressure, arrhythmias, and bradycardia.
  • Bulbar Palsy Monitoring: Assess for difficulty swallowing, as this indicates a high risk of aspiration.

2. Definitive Care (Immunomodulatory Therapy):

  • Treatment should be started in patients who are unable to walk independently.
  • There are two proven, equally effective options:
    • Intravenous Immunoglobulin (IVIg): A 5-day course. It is generally the preferred option due to ease of administration.
    • Plasma Exchange (PLEX): Involves removing the patient’s plasma (which contains the pathogenic antibodies) and replacing it with a substitute.
  • Corticosteroids have been shown to be ineffective and should not be used.

3. Supportive Care and Rehabilitation:

  • Prevent Complications: Prophylaxis for DVT (with heparin), management of pain (neuropathic pain is common), and prevention of pressure sores.
  • Rehabilitation: Early involvement of physiotherapy and occupational therapy is crucial for a long recovery period.
What is the CSF abnormality seen in patients with GBS?

A lumbar puncture is performed to support the diagnosis. The CSF analysis often shows a characteristic pattern, though it may be normal in the first week of illness.

  • The typical finding is Albuminocytologic Dissociation.
  • This means:
    • High Protein Level: The CSF protein is elevated (due to inflammation of the nerve roots).
    • Normal Cell Count: The white cell count in the CSF is normal or near-normal.

This pattern helps to differentiate GBS from infectious causes of weakness, which would typically have a high white cell count (pleocytosis) in the CSF.

Adult Medicine Long Case: A Young Patient with Stroke

Patient Summary

A 45-year-old female presents with a sudden onset of left-sided weakness and facial droop which started 2 hours ago. She has no significant past medical history, is a non-smoker, and is not on any regular medication. There is no history of recent trauma, headache, or neck pain. On examination, her blood pressure is 140/90 mmHg. Neurological examination confirms a dense left hemiparesis and an upper motor neuron left facial palsy. Her NIHSS (National Institutes of Health Stroke Scale) score is 12.

How would you manage this patient in the acute stage?

This is a hyperacute stroke presentation. The management is extremely time-sensitive, following the “Time is Brain” principle and a structured stroke protocol.

  1. Immediate ABCDE Assessment & Pre-hospital Alert: Alert the nearest hyperacute stroke unit immediately with a “Stroke Alert” call. My initial actions are to ensure a patent airway, adequate breathing, and circulation. I would check a capillary blood glucose to urgently rule out hypoglycaemia, a key stroke mimic.
  2. Rapid Transfer to Hospital: The patient needs immediate transfer to a hospital with 24/7 CT scanning and a hyperacute stroke team.
  3. On Arrival at the Emergency Department:
    • The stroke team should meet the patient on arrival.
    • A rapid, focused history and neurological examination (including NIHSS scoring) is performed.
    • Secure IV access and send baseline bloods (FBC, U&Es, coagulation screen, group and save).
    • The patient is transferred directly to the CT scanner. This is the most critical first step in the hospital.

The entire process from door to imaging should ideally be less than 20 minutes. The goal is to make a decision about reperfusion therapy as quickly as possible.

Discuss the role of imaging in acute stroke in detail.

Imaging is the cornerstone of hyperacute stroke management, used to diagnose, guide therapy, and predict prognosis.

  • Non-Contrast CT (NCCT) Brain:
    • Primary Role: The most important reason to perform an immediate NCCT is to exclude intracranial haemorrhage. The clinical presentation of ischaemic and haemorrhagic stroke can be identical, but their treatments are opposite. Giving thrombolysis to a patient with a bleed is catastrophic.
    • Secondary Role: It can also identify stroke mimics (e.g., tumour, abscess). In the first few hours of an ischaemic stroke, the NCCT may be normal. However, early signs of a large infarct may be visible, such as loss of grey-white matter differentiation or a hyperdense MCA sign (a clot in the middle cerebral artery).
  • CT Angiography (CTA) of the Head and Neck:
    • Role: This is now performed routinely at the same time as the initial NCCT in most stroke centres. It involves injecting IV contrast to visualize the arteries.
    • Purpose: Its primary purpose is to identify a Large Vessel Occlusion (LVO) (e.g., in the internal carotid or middle cerebral artery), which would make the patient a candidate for mechanical thrombectomy. It can also identify other causes like arterial dissection.
  • CT Perfusion (CTP):
    • Role: CTP is an advanced imaging technique that assesses cerebral blood flow. It can identify the irreversibly damaged “ischaemic core” and the surrounding “penumbra,” which is brain tissue at risk but still salvageable.
    • Purpose: It is crucial for selecting patients for mechanical thrombectomy in the extended time window (6-24 hours).
  • MRI Brain: While MRI (specifically the DWI sequence) is more sensitive for detecting acute ischaemia, it is less used in the hyperacute setting due to longer scan times, limited availability, and difficulties in monitoring unstable patients.
Discuss reperfusion therapy in detail, including the full inclusion and exclusion criteria for thrombolysis.

Reperfusion therapy is the definitive treatment for acute ischaemic stroke.

1. Intravenous Thrombolysis with Alteplase (rtPA):

Inclusion Criteria:

  • Clinical diagnosis of ischaemic stroke causing a clear, measurable neurological deficit.
  • Onset of symptoms < 4.5 hours ago (the earlier the better).
  • Age > 18 years.
  • Non-contrast CT brain shows no evidence of haemorrhage.

Absolute Exclusion Criteria:

Category Specific Contraindications
Evidence of Bleeding
  • Any current or past history of intracranial haemorrhage.
  • Active internal bleeding.
  • Known intracranial AVM or aneurysm.
High Risk of Bleeding
  • Major surgery, serious head trauma, or previous stroke in the last 3 months.
  • GI or urinary tract haemorrhage in the last 21 days.
  • Arterial puncture at a non-compressible site in the last 7 days.
  • Known coagulopathy: Platelets < 100,000, INR > 1.7, or current use of anticoagulants (with elevated lab values).
  • Persistently elevated blood pressure (Systolic > 185 mmHg or Diastolic > 110 mmHg) despite treatment.
Clinical/Radiological Factors
  • Symptoms are minor or rapidly improving.
  • Clinical presentation suggestive of subarachnoid haemorrhage.
  • CT shows evidence of an established, large infarct (hypodensity > 1/3 of the cerebral hemisphere).
Source: AHA/ASA guidelines for the early management of acute ischemic stroke (2019).

2. Mechanical Thrombectomy:

  • Indication: For patients with an acute ischaemic stroke caused by a Large Vessel Occlusion (LVO) in the anterior circulation.
  • Time Window: Can be performed up to 24 hours from symptom onset in eligible patients selected via advanced imaging (CT Perfusion or DWI-MRI).
How would you investigate the aetiology in this young stroke patient?

A stroke in a young patient (<50) without traditional risk factors necessitates a broad and thorough investigation to find an uncommon cause.

1. Detailed Cardiac Evaluation (to exclude cardioembolism):

  • Prolonged ECG Monitoring: A 24-hour Holter monitor is a minimum. Often, longer monitoring (e.g., 7-day tape or implantable loop recorder) is needed to search for paroxysmal atrial fibrillation.
  • Echocardiogram with Bubble Study: A transthoracic (TTE) and often a transoesophageal (TOE) echo is performed to look for:
    • Patent Foramen Ovale (PFO) with atrial septal aneurysm: A key cause of cryptogenic stroke. The bubble study looks for right-to-left shunting.
    • Ventricular thrombus, valvular disease, or infective endocarditis.

2. Detailed Vascular Evaluation:

  • CT or MR Angiography of the Head and Neck: To look for:
    • Arterial Dissection: A tear in the wall of the carotid or vertebral arteries. A key cause in the young, sometimes associated with neck pain or minor trauma.
    • Vasculitis or Vasculopathy: Such as fibromuscular dysplasia.

3. Detailed Haematological Evaluation (Thrombophilia Screen):

  • This is a comprehensive blood panel to look for a hypercoagulable state. It is particularly important if the stroke is cryptogenic or if there is a history of other thrombotic events or pregnancy loss.
  • Tests include:
    • Antiphospholipid Syndrome screen: Lupus anticoagulant, anti-cardiolipin antibodies, and anti-beta-2-glycoprotein I antibodies.
    • Inherited thrombophilias: Factor V Leiden, Prothrombin gene mutation, Protein C, Protein S, and Antithrombin deficiencies.

4. Other Investigations:

  • Autoimmune Screen: ANA, ENA, and ANCA to look for systemic inflammatory diseases like SLE or vasculitis.
  • Genetic Testing: In rare cases with a strong family history, testing for conditions like CADASIL may be considered.
  • Toxicology Screen: To look for prothrombotic drugs like cocaine or amphetamines.


Leave a Reply


Latest Posts